You are on page 1of 155

1

1. Best retentive cement for cr


Resin* Retention =resin
Reinforced GI*
Zn polycarboxylate
2. Bacteria that initiate & propagate caries →strept coccus mutans& lactobacilli
3. After many details...he asks about disinfectant materials used for impression
formaldhyde
Acetaldehyde

4. Access cavity for lower mand. 6


a. Rhomboid
b. triangle e' it's base distally
c. trapezoidal****
2

5. Picture of x-ray and ask about type of caries→root caries pic.is very obvious
6. Pic. of white multiple small lesion at the highly red soft palate, the Q. say that pt.
undergo a transplanta on from 2 months
a. pseudo candidiasis****
b. erythematous candidiasis

7. deficiency of V B12...
a. macrocytic anemia
b. microcytic anemia
c. pernicious anemia*****
8. can't remember accurately...but he
asked about the relation bet. blade of
the instr. and its handle
a. parallel***
b. perpendicular...
9. fluoride varnish mostly used...
a. stannous fluoride 5%
b. St.fl..2%
c. Na F 5%***
d. Na F 2%
10.water irrigation system... ‫ﻣش ﻓﺎﻛرة ﺑﺎﻟظﺑط ﻛدا او اﯾﮫ‬
a. to prevent plaque formation
b. to decrease plaque formation
c. for dilution of saliva containing bacteria ***
3

11.Q about dentin &hybrid layer...etched dentin

12.in deciduous teeth after complete pulp exacerbation we use


a. dycal
b. Zn oxide E cement***
13.Q about the mat. that.... caries→propylene glycol
14.Q about teacher Collin syndrome and characteristic feature of it→ malar bone
defect
15.Q about Schick test →Hypersensitivity of diphtheria
16.19..22y pt. who recently stop smoking, has ulcers on gingiva, I didn't remember
choices but the Q very easy.... ANUG
17.20- old pt. has some upper molars overerupted While lower jaw ant. have gingival
recession, degree of mobility & Kennedy Cl 1
a. acrylic RPD e' occlusal adjustment of upper over erupted teeth
b. swing lock also e' adjustment =need too much mobility
c. over denture ***
4

18.21-female pt. 55 y has "shams"(= Arabic word l think it's similar to pedunculated
nevus) in her check. she used to bite on it for several years, recently she has a
complain (but I forget w is). Q enumerate some histologic feature related to
cancerous lesion
a. SCC
b. verrocous carcinoma
19.22-temporal muscles
innervated by.
a. Facial
b. auriculotemporal
c. deep temporal n*****
20.In perio. Surgery we wait 6
months for what→maturation
21.24-pt. e' suppurative swelling related to lower 6, tender tooth, -ve e' vitality test
a. incision& drainage
b. debridement, incision& drainage ****
c. give him ttt
22.25-considered one of disadvantages of steam sterilization
a. long time
b. dulling of non-protected burs ****
23.26-Q about incision & drainage for abscess related to lower 6
a. open at the most fluctuant point***
b. no need for large opening if the pus already begin to extrude
c. opening at the end of gingival sulcus
d. incision until reach the periosteum
24.27-most used Barker blade
1.15****
2.10
3.12
25.child with bilateral missing D and the first permanent molars are completely
erupted what type of space maintainer??
Bilateral band and loop***
Lingual arch
Distal shoe
26.Bacteria initiated the caries
→Streptococcus mutans
27.Steps of sterilization→
28.Type of cement used in class 5→GIC
5

29.Type of bacteria cause endocarditis


a. Aerobics *** =Staph. Aurues & strept. viridians
b. Anaerobic mixed

30.Long story about pt. comes with severe pain a er 3 days of amalgam fillings the
filling was good but there was composite filling in the opposite tooth
a. Change the filling
b. Put varnish ***
c. Extract the tooth
31.X-rays used to detect disk perforation
a. MRI
b. Arthrography***
c. Reverse town
32.U-shaped radiopaque line in x Ray
a. Zygoma ***= 12 Body of the zygoma c bone, with
innominate line
b. Zygomatic process 18
c. Maxillary process
33.Cause of isthmus fracture
a. High filling *
b. Wide isthmus

34.Type of Florid given to mentally retarded pt.


a. Sodium fluoride
b. Varnish fluoride**
35.Mobility of mid face with step deformities at fronto zygomatic suture
a. Lefort 1 fracture
b. Lefort 2
c. Lefort 3****
36.Strange question with diagram cell A positive sample The row shows the truth and
the column show the test results
37.The rows show " truth “, the column show " test result " 0
A - cell A has true positive sample. ***
B- cell A has true negative sample.
C - cell A has false positive sample.
6

D - cell A has false negative sample.

38.Instrument used to remove fractured parts from pocket →Schwartz periotriever


39.Pulp stone all true except
a. Cause pain **
b. Free in pulp chamber
7

40.Most common tooth introduced to maxillary sinus

41.Pulp infection in primary teeth less than permanent why →


More blood supply in primary
42.Best AB to pregnant women
a. Penicillin***
b. Ampicillin
c. Clindamycin
43.Palate composed of
a. Palatine and sphenoid bones
b. Palatine and maxillary bone ****
c. Palatine and zygomatic bone
8

44.Difference between gracey and universal curette

45.What is the forceps used to grasp epulis fissuratum during surgery?


a. Alias***
b. Addison
c. Curved hemostat
9

46.Differentiate between pulp and periodontal abscess


a. X-rays
b. Vitality test****
c. Thermal
d. Percussion

47.Management of fractured root between middle and apical third


a. Rct for coronal+ surgical removal of apical ****
b. RCT for apical and remove coronal part
c. Splint

48.Healing after scaling and root planning accomplished by


a. Long junctional epithelium ****
b. Bone and connective tissue migration
10

49.Long story about pt. with complete denture come with inflammation of upper
buccal vestibule → Hypertrophic frenum
50.All are true about dentinogenesis imperfecta except
a. Blue sclera =in type I
b. Multiple bone fractures =in type I
c. Thin enamel and dentine
d. Supernumery teeth****

51.Radiopaque mass at the apex of the mandibular first molar which is separated
from the bone by radiolucent margin→ Cementoblastoma
11

52.Osteogenesis in endodontic surgery aimed to →Prevent fibrous tissue growth


53.How increase retention in amalgam cavity
a. The walls divergence occlusally
b. The walls converge occlusally ***
c. Parallel
54.Least accurate test
a. Thermal  least accurate= electric
b. Electric ***  most accurate= thermal
c. Cavity.  accurate but invasive= cavity
55.Inferior alveolar n branch of
a. Anterior branch of mandibular n
b. Posterior branch of mandibular n****
c. Trunk of mandibular n
56.Location to give inferior alveolar
nerve block
a. Pterygomandibular raphe
b. Coronoid notch
c. All****

57.Pt with complete denture pronounced f as v


a. Maxillary anterior teeth had placed too far from superiorly and
anteriorly ***
b. Anterior teeth are upward from lip line
12

58.To enhance strength property of ceramometal restoration it's important to


a. Avoiding sharp or acute angle
b. Build up a thick layer of porcelain
c. Porcelain should have uniform thickness
d. Compensation any defects in the preparations equally by porcelain and metal
A and B
A and c *****
B and d
59.What is the time between the first onset of HIV and appearance of acute
symptoms?
a. 1-5y
b. 9-11****
c. No specific time
60.Periodontal ligaments in middle third of root
a. Oblique ****
b. Transseptal
c. Horizontal =coronal 1/3
61.Child with previous history of minor Trauma
with excessive bleeding we do test the result
is prolonged PT and slightly increased
clothing time diagnosis is
a. Hemophilia b
b. Thrombocytopenia
c. Vit K deficiency**** ‫ﻧﺑﯾل‬
62.Pt come to clinic with ill-fitting denture
during examination your noticed white small
elevations on the crest of lower ridges what will tell the pt.→The pt. should not
wear the denture for 2 weeks then follow up
63.The way to know if there's two canals →Radiography with 2 files inside the roots
13

64.Cause of protaper file fracture

65.Question about HIV pt. and bleeding happen during dental treatment how you
should do !!→
put plaster and asked the pt. to make pressure on the wound and send him to
specialize
66.1- child presents with bilateral swelling of submandibular area, what could be the
disease:
a. Fibrous dysphasia. = unilateral
b. Cherubism ***
c. Polymorphic adenoma
d. ameloblastoma

67.2-Medicine ethics aim to:


a. The dentist should study to know patient psychology.
b. Not to compromise or undermine ability to treat patient in community as
professional.
c. Ability to make decision.
d. All of above. ***
68.3-Which of the following non- absorbable suture: → Silk. ***
14

69.4-Diabatic pt. with multiple nevi on the neck and the scalp, and multiple jaw cysts,
ur diagnosis will be: → Gorlin-Goltz syndrome. ***

70.5-The body secrets antibody against antigen using which cells: →a. B lymphocyte

71.6-Acyclovir dose for treatment of herpes: →200 mg / 5 mes a day.

72.7-Choice of local anesthesia technique influenced by:


a) Chemical composition of anesthesia.
B) The location of the nerve
C) Bone structure. ***

73.8-20 years old pt. have avulsed tooth for 60 min. the management to return
vascularity of the tooth:
A. Scrap the surface of the root.
B. Place the tooth in sodium sulfide of X %.
C. soaking in 2.4% sodium fluoride*****
15

74.Edentulous pt. class II kennedy classifica on 2nd premolar used as abutment


when we serving we found mesial undercut what is the proper clasp used:
1/ wrought wire with round cross sec on
75.9- Bitewing x-Ray is used to diagnose all EXCEPT:
1. Proximal caries.
2. Secondary caries.
3. Gingival status.
4. Periapical abscess *****

76.10- In endo, one of sealer property is to be flowable (or wettability) to enhance


this quality we can mix it with a material that have:
a. Low surface tension***
b. High surface tension
77.11- You extract tooth with large amalgam restoration, how to manage the
extracted tooth:
1. Autoclave and deep buried.
2. Sharp container.
3. Ordinary waste container
16

4. Office container
5. Container Designed not to be Incinerated. ****
78.12-Amalgam filling opposite gold onlay and cause pain, he mentioned the
amalgam filling is so good what should you do:
1. Change the restora on
2. Extrac on.
3. varnish apply. ***
4. apply medium separator

79.13- Forceps to hold flap when suturing: →A. Adson

80.14-we want to do
a maxillary PD to a
patient using
anterior-posterior
strap, we want it
to be rigid, we
use:
b- gold.
c- cast gold.
d- co-cr. ***
(cobalt-chromium)
17

81.15-Solitary bone cyst management:


a. Anti-inflammatory and follow up.
b. Curettage and close. ***
c. Marsupialization and antibiotic
d. No active management

82.16-In terms of caries prevention, the most effective and most cost effective
method is:
a. Community based programs. *** (Artificial water fluoridation ).
b. Individually based programs.
c. Private based programs
83.17-AH26 is root canal sealer consists of
a. ZOE.
b. Epoxy resin. ***
c. Steroids
d. all of the above
84.18-How can u repair have fractured rest (in the
place where it passes over the marginal ridge of
the tooth) in
removable partial
denture?
A- Spot welding
c- Industrial brazing.
b- Electric
soldering****
18

85.19-Central incisor receiving a full ceramic restoration during finishing of shoulder


finish line subgingivally, what is the type of preferred bur: →Diamond end cutting
86.20-Post graduate student uses MTA the prognosis depends on prevent:
a. immediate suture.
b. disturbance during closure of wound. ***
c. using a flab.
87.21-Which one is true and which one is false:
a. Systemic factors can cause periodontal conditions: false
b. Systemic factors can modify periodontal conditions: true
88.22-Patient with asthma on chair,
U give subcutaneous epinephrine for management what is the dose:
A. 1:100
B. 1:1000****
C. 1:10000.
19

89.23-The least effective method to kill the HIV is through:


A. Sodium hypochlorite "NaOCL". very effective
B. Ultraviolet chamber. *** (Ultraviolet light).
C. Autoclave.
D. Chemoclave
90.24-Best way to detect presence of 2 canals:
a) Radiographically with 2 files inside the root. ***(pu ng 2 files & take X-ray)
b) The orifices are close to each other.
91.25-Patient comes to you with edematous gingiva, inflamed, loss of gingival
contour and recession, what's the best tooth brushing technique?
A. Modified bass.
b. Modified stillman. *** ‫اﻟﻣﻠﺧص‬
c. Charter.
d. Scrub
92.26-Patient complains from pain in TMJ. During examination you noticed that
during opening of the mouth mandible is deviate to the right side with left
extruded. Diagnosis is:
a) Condylar displacement with reduction.
B) Condylar displacement without reduction.*
93.27-Patient with complete denture pronouncing F as a V:
20

94.28-Best type of core material for post→Amalgam


95.Patient has a palatal torus between hard & soft palates, the major connector of
choice: →-U shaped. (horseshoe).
96.Patient has a palatal torus in hard palate the major connector of choice:
- Anterior-posterior palatal strap
98.1Q about cavo surface angle for amalgam

99.2-limitation of usage of apex locator


a. narrow canal
b. short canal
c. curved canal
d. open apex***
100. 3 -q about bacteria that cause failure of endo
a. staph
b. strep
c. feacalis***
101. 4- fluctuant abscess and u decide to drain it which blade u will use
21

a. no 11***
b. no 15
c. no 12
102. 5-aggressive usage of brush
will cause
a. abrasion ***
b. corrosion
c. erosion
103. 6- by x ray u found apex of the root of max. first molar fused with root of 2nd
max. molar ... name of this case→Concrescence

104. 7- by radiometry u found


the halogen light cure is 450
what u will do
a. change battery
b. change the lamb
c. don't make Any
change**** because
it is inside normal
range

105. 8 pics of upper canine


with deep caries and
radiolucency attached to the root
a. granuloma ***
b. radicular cyst
c. periapical abscess
d. Cemental dysplasia
If small granuloma, big abscess, big with well-defined margin cyst
22

106. 9 pt. has missed lower 4 ants. and with slight class l mobility of 2 canines and he
wanted to partial denture what the type of major connector ‫اﻟﺳؤال ﻣﻌﺎه ﺻورة و ﻓﻲ‬
lower 6 ‫ →ﻣﺧﻠوع‬lingual bar

107. 10 q about func on of


irrigation system
a. remove dental plaque
b. decrease plaque
c. dilute bacteria****

108. 11 when u use


Supraperiosteal inflt.
a. when there is infection
b. in dense bone
c. in mandibular inflt.
23

d. soft tissue anaeth. In surgery

109. 12 q about the difference between gracey and universal curette

110. 13 the most economic imp u will use when the rest of metal partial denture is
broken and u need to take overall imp with high details
a. Alginate
b. poly vinyl****
c. poly ether
d. don't remember
111. 14 old pat 55 y with xerostomia and come with this case (pic of canine)
a. root caries ***
b. abrasion
c. erosion
24

112. 15 bacteria that not found in new infant→Strep mutans


113. 16 bacteria that initiate caries→ Strep mutans
114. 17 tuberosity block tech is
a. Gow gates
b. Akinosi
c. post super. Alv***

115. 18 q about growth center between 2 bones in the skull→ Sphenooccipital


synchondrosis
Synchondrosis (hyaline cartilage lined with periosteum on both sides with
hyperplastic cells in middle) Between: - Intersphenoidal, Sphenooccipital & Spheno
ethmoidal
116. 19 what is the func onal cusps in man and max.→
Max: palatal/lingual cusps
Mand: buccal cusps
117. 20 the distance between 2 implants→ 3mm
118. 21The ttt of herpes simplex →200 mg / 5 mes a day.
25

119. 23 q about the bone Rongeur


120. 24The name of first brachial arch→Mandibular
121. 25what is the fracture that involves the enamel and dentin and cementum
a. crown root fracture ****
b. crown fracture
122. 26 fracture in the middle of the root and the tooth is symptoma c what is the
ttt
a. extract the tooth
b. endo ***
c. extract apex

d. leave without ttt


123. 27 pic of x-ray and the patient suffered from swelling and pain before‫ ﻗﺑل‬eating
a. sialothisis ****
b. pain from Impacted 8
c. pain from Fpd
124. 28 pat has swelling from the sub lingual gland and u decide to make incision
where do you will make the incision? →If the space contains pus, the usual
treatment is by incision and drainage. The site of the incision is intra-oral, made
lateral to sublingual plica. OR Intraorally parallel to Wharton's duct
125. 29 q about the size of GG # 1 →50****
26

126. All r muscles of


expression except
a. zygomaticus
b. masseter ****
c. orbicularis
d. buccinator
127. Pat has a sensitivity to
amide type of anaeth.
And u decide to use ester
type ‫ أظن‬prilocaine ‫ﺗرﻛﯾز‬
‫ﻣش ﻓﺎﻛر‬. what is the color
of the carpule? → Answer
is
a. Yellow = prolicaine 4%
with epinephrine
1:200 000 OR
b. Black = prolicaine 4% plain
128. pt. Come for surgical extraction of impacted lower wisdom & he is under
warfarin therapy What is the correct INR level at the day of surgery for dentist
to do the surgery:
a. PTT= 1-1.5
b. PTT =2-2.5
c. PT= 1-1.5*******(this is the answer trust me plz not as the files)
d. PT 2-2.5
27

129. About standardization of the file refer to what?

Standardization of Intracanal instrument


-Originally, it suggested by Ingle.
-The American Na onal Standardiza on ins tute(ANSI) specifica on No. 28 for K-
type Rc. file: -
1-The position at which the cutting blades begin on the inst. called D◦
2-The position at which the cutting blades end on the inst. called D⑯
3-The length of inst. from D◦ to D⑯ = 16mm =Length of Working part.
4-Overall Length from the p to the handle =21,25,28 or31mm.
5-Diameter at D◦ is smaller than D⑯ by 0.32mm.
6-There is a constant taper of 0.32/16=0.02mm diameter/mm length.
7-The p should be 75ºor+/-15º
8-The numbering system of the inst. is based on the Diameter of the inst.
Size 60 at D◦ =60/100=0.6mm
9-There must be Gradual incremental increase in size at D◦ from one inst. to another
a. by 0.02 mm between sizes 6,8,10.
b. by 0.05 mm from size 10-60
c. by 0.10 mm from size 60-150
10-Inst. of medium sizes: -
 In Narrow curved Rc.
i. Sometimes, it is difficult to go with the next larger size to the exact working
length →Cu ng of 1mm from the p of the inst.→Intermediate sizes.
ii. Example, ●F15 has D◦ =0.15mm
● Cu ng one clip of 1mm from the p →
D◦ =0.15+0.02=0.17mm=intermediate size between size 15 & 20.
The intermediate sizes are now available.
11-Profile series 29:-It is a new concept in inst. design
In the current ISO standard, the difference in size between files at Dº could
reach 50%.
Example,
 a-the difference between size 10 &15 at Dº=
Increase in size from size 10 to 15 0 . 05
%  %  50 % , this means
Diameter of file 10 at D 0 .1
between 2 successive files there is a rapid increase in size→Difficulty in
using of F15 following F10 in the Rc.
b-the difference between size 55&60 at Dº=
Increase in size from size 55 to 60 0 . 05
%  %  9%
Diameter of file 55 at D 0 . 55
28

130. 1- What is the tooth surface mostly suscep ble to caries at age of 9?
a- labial of upper canine
b- occlusal of lower first molar***
c- mesial of upper premolar
d- occlusal of upper premolar
131. 2- The space between upper B and C is called:
a- leeway space
b- primate space***
c- freeway space
29

132. 3- What is the impression material the can record fine details even on wet
surfaces?
a- polyether***
b- polysulphides
c- condensation silicone
d- addition silicone

133. 4- Proxy brush is used with which type of embrasure?


a- type 1
b- type 2**** & grade III furcation
c- type 3
d- type 4
134. 5- A photo like this one and asks

about Kennedy Applegate


classification of lower ridge:
a- class 3
b- class 1 modifica on 1****
c- class 4
d- class 2 modifica on 1
30

135. 6- The least important to check during complete denture try in:
a- vertical dimension
b- adaptation of margins
c- retention
d- centric relation*** =Bcz bone to bone relation
136. 7- C shaped canals are often seen in:
a- maxillary first premolar
b- maxillary second molar
c- mandibular first molar
d- mandibular second molar***
137. 8- Indirect retainer in RPD is characterized by:
a- very thin
b- makes right angle with major connector****
c- makes acute angle with major connector
d- very retentive (engages into undercuts)
138. 9- A photo like this one and asks about the name of the technique:
a. Akinosi = Akinosi technique, closed-
mouth mandibular nerve block,
tuberosity technique
b. inferior alveolar nerve block
c. gowgate = mandibular nerve block****
d. incisive nerve block
139. 10- The common concentration of Fluoride in
over the counter dentifrices in
ppm is:
a- 230 ppm
b- 600 ppm
c- 1000 ppm***(den frice: 1000
ppm, mouth wash: 223 ppm)
d- 2300 ppm
140. 11- What technique will you use
to anesthetize soft and hard
tissues of mandibular molars with
one injection?
a. Akinosi
b. inferior alveolar nerve block
c. gowgate = mandibular nerve
block****
d. incisive nerve block
31

141. 12- You gave a patient an inferior alveolar nerve block to make some
restora ons. He returned to you a er 2 days with fever, pain and swelling in the
same side. What is the
space infection he is
suffering from?
a- submandibular
space
b- pterygomandibular
space***
c- lateral pharyngeal
space
d- masseteric space
142. 14- Fluoride varnish used commonly is:
a- 2% stannous fluoride
b- 2% sodium fluoride
c- 5% stannous fluoride
d- 5% sodium fluoride****

143. 15- Vasoconstrictor role in endo-surgery is:


a- inhibition of A fibers in bone
b- inhibition of A fibers in periodontal ligament
c- decrease toxicity of L. A
d- increase duration of L.A****
32

144. 13- how to safely accelerate the setting of alginate?


a- increase powder water ratio
b- increase mixing water temperature***
c- add accelerator powder
d- you can't accelerate setting reaction
1. Water Temperature……. Warmer water will speed the setting time of
alginate and cooler water will slow it. (For a graph of setting times vs. water
temperature, look below.
2. Mixing Ratio……. Thicker alginate sets faster, thinner sets slower- all else
being equal.
3. Air Temperature……. Air temperature will affect how quickly the alginate
will cool during and after mixing- same “warmer/faster, cooler slower”
effect, however layup molds will be much more affected because of the
larger “surface area to volume” ratio. Spread thin, it will be more affected by
air temperature and evaporation than a bucket mold.
4. Humidity……. Since evaporation off the surface of the alginate will cool the
mass, lower humidity will increase evaporation, cooling the mix and slowing
the set. Higher humidity will lessen evaporation, allowing the alginate to
stay warmer which will shorten the setting time.
5. Chemicals in the water……. In general, our recommendation is, “If you feel
fine drinking your tap water, then it’s OK to use to mix your alginate”. If you
filter your drinking water or there are noticeable odors from your tap water,
best to use bottled water. The cheap stuff is fine. We have no idea what
effects your tap water might have on the setting time or any other working
145. 16- property
X-ray likeof
this onealginate
your and asks about this
formation:
a- compound odontoma****
b- complex odontoma
c- concrescence
d- adhesion
N.B.: If the radiopaque structure clearly resembles
dental structure, then it is compound odontoma. If
not or poor resemblance, then it is complex
odontoma.
146. 17- Change in chemical indicators means that
the load is:
a. Sterile****
b. Disinfected
c. exposed to hot steam
d. incinerated
33

147. 18- the location of finish line in abutment tooth is determined by:
a- opposing teeth
b- alignment of the neighboring teeth
c- length of clinical crown****
d- type of planned restoration
148. 19- Types of steel used in many dental
instruments:
a- stainless steel and tungsten steel
b- stainless steel and cobalt steel
c- stainless steel and carbon steel***
d- carbon steel and tungsten steel

149. 20- How to verify the success of ttt of apical periodontitis:


a- decrease in the size of radiolucency
b- fistula closure
c- the tooth becomes asymptomatic ****
d- negative pulp test
34

150. 21- During evaluation of the periodontal condition of abutment teeth, what has
the most importance?
a- horizontal bone
b- vertical bone
c- equally important ****
d- not important
151. 22- when should the child visit the orthodontist for the first time?
a- when upper permanent incisors erupt***
b- when all deciduous teeth erupt
c- in case of spacing
d- in case of crowding

152. 23- most often used surgical blade:


a- number 11
b- number 12
c- number 15****
d- number 16
153. 24- most conservative procedure during occlusal analysis for RPD:
a- onlay preparation
b- extraction
c- spot grinding ****
d- crown preparation
154. 25- most important consideration before construction of functional appliance:
???
a- skeletal age*
b- sex of the patient
c- type of appliance
d- erupted and non-erupted teeth
35

155. 26- A patient lost his gagging reflex, what cranial nerve is affected?
a- VII
b- IX
c- X
d- XII
156. 27-

During clinical examination prior to complete denture construction, the dentist


asked the patient to say (Ahhh); he noticed that the left half of soft palate is not
moving. That indicates the injury of cranial nerve:

a- VII
b- IX
c- X***
d- XII
157. 28- what are the teeth most susceptible to fluorosis:
a- mandibular incisors
b- maxillary molars
c- maxillary premolars*** = Teeth that calcified slowly cupid bicuspid.
d- mandibular molars
36

158. 29- what does the last number in the four numbered formula of a periodontal
instrument refer to?
a- angle of cutting edge
b- angle of the blade***
c- width of the blade
d- length of the blade

159. 30- Most common salivary gland tumor:


a- pleomorphic adenoma ****
b- adenoid cystic carcinoma
c- mucoepidermoid carcinoma
d- actinic cell carcinoma
160. 31- Critical PH of saliva at which
enamel starts to demineralize:
a- 4
b- 5.5****
c- 7.5
D- 12.5
161. 32- Type of orthodontic force
applied to teeth with
compromised attachment level:
a- tipping
b- light***
c- heavy
d- intermittent
162. 33- What is the name of the condition at which both rate and depth of respiration
increase?
a- hypoventilation
b- hyperventilation***
c- Cheyne-Stokes
d- hypoxia
163. 34- If a patient you
treat had a hyperventilation attack; what will you do?
a- give him Oxygen
b- don't do anything
c- ask him to breath in a bag***
d- call the ambulance
164. 35- A 22 years old female pa ent just removed her fixed orthodontic appliance
and complains from white spot a labial surfaces of the anterior teeth.
How to make these white spots disappear?
37

a- night guard bleaching


b- walk bleaching
c- microabrasion***
d- composite veneer
165. 36- Lengths of files and reamers:
a- 22-26-30
b- 23-27-31
c- 25-28-33
d- 21-25-31****
166. 37- What is a crater?
a- one wall defect
b- two walls defect***
c- three walls defect
d- gingival recession

167. 38- during examina on of 44 years old male you found some teeth with gross
occlusal caries; you want to check the proximal surfaces of his molars. What
scanning type you will use?
a- periapical radiograph
b- occlusal radiograph
c- CBCT
d- bitewing radiograph*****
38

168. 39- A 7 years old boy with a habit of thumb sucking, what type of ttt?
a- counseling therapy***
b- adjunctive therapy
c- rewarding therapy
d- wait
169. 40- Concentration of
chlorhexidine in
commercial mouthwash is:
a- 1%
b- 1.2%
c- 12%
d- 0.12%****

170. 41- We use discoid cleoid


instrument for:
a- burnishing
b- carving***
c- condensation

d- polishing
39

171. 42- The angle between the blade and facial surface during scaling should be:
a- 25-50
b- 45-90***
c- 100-110
d- 90-130
172. 43- Treatment of acute Pericronitis:
a- antibiotic only
b- extraction and antibiotic
c- rinse and clean****
d- no ttt

173. 44- The part of X-ray machine that reduces the dose of X-ray:
a- Anode
40

b- Cathode
c- collimator ***
d- cone
174. 45- You will construct a class II RPD and found that the most distal abutment has
a Buccal undercut. What clasp design you will include for this abutment????
a- RPA
b- RPI***
c- circumferential***
d- rigid
175. 47- What is the suitable temporization for anterior full crown preparation?
a- aluminum sheet
b- silicate cement crown***
c- stainless steel crown
d- zinc oxide eugenol
176. 46- You are doing clinical examination for maxillary edentulous ridge and found a
severe undercut at the maxillary tuberosity. What should the surgeon do?
a- use blade number 15 to displace the distal end of the tuberosity
b- make Buccal and palatal slopes from the tuberosity
41

177. 48- Gutta-percha is composed mainly of:


a- Gutta-percha
b- resin
c- filler
d- zinc oxide****
178. 49- Contraindication for porcelain laminate veneer:
a- fluorosis
b- diastema
c- high lip line
d- bruxism*****
42

179. 50- What is the reason of gingivectomy procedure for patient with Dilantin-
induced gingival hyperplasia?
a- elimination of dental plaque
b- elimination of bony defect
c- elimination of pseudopocket***
d- elimination of granulation tissues
180. 51- What do you call the procedure of sacrificing one root of a multirooted tooth
to preserve the integrity of remaining tooth structure?

a- root resection
b- root amputation****
c- Apicectomy
d- root end resection

181. 52- How to detect facial


asymmetry?
a- posteroanterior radiograph***
b- CBCT
c- Orthopanthomgraphy
d- lateral cephalometric

182. 53- A photo of a male with completely edentulous maxilla and missing
mandibular six anteriors. The pa ent is a 45 years old controlled diabe c with no
mobility or bone loss. He said that he never got an appliance before.
What is the proper prosthesis for lower jaw?
a- FPD
b- Acrylic RPD***
43

c- Implants
d- Metallic RPD
183. 54- What is the appropriate treatment plan for a physically challenged 5 years old
boy with multiple carious lesions?
a- wait until all teeth become destructed then extraction
b- full ttt under L.A.
c- full ttt under G.A.***
d- fluoride application
184. 56- What is the oral manifestation of a patient maintained on cyclosporine?
a- mucositis
b- gingival enlargement***
c- xerostomia
d- no oral changes
185. 57- What will increase with overcontoured crown?
a- esthetics
b- periodontal health
c- masticatory efficiency
d- plaque score***
186. 58- What is the easiest tooth for RCT?
a- overfilled
b- underfilled
c- tooth with weeping canal***
d- tooth with silver points
187. 59- What is the material that can reduce discoloration of amalgam?
a- cavity varnish***
b- dycal
c- zinc phosphate cement
d- zinc polycarboxylate cement
188. 60- Appropriate time to treat a patient on hemodialysis:
a- same days of hemodialysis
b- a er 2 days
c- a day after**
d- postpone t for 2 weeks
189. 61- The main reason for using floss before application of rubber dam:
a- remove entrapped food
b- verify the tightness and roughness of the contact area***
c- open contact between teeth
44

d- ligate rubber dam to tooth = after application

190. 62- During extraction of upper third molar; the tooth suddenly slipped and
disappeared, what should you do?
a- extract a er 3 weeks
b- make CT and extract under GA***
c- follow up
d- do noting
191. 63- Most important property of sealant:
a- viscosity
b- retention***
c- resilience= for RC. sealer
d- strength
192. 55- 70 years old male pa ent asks to construct a complete denture. He shows
you three CDs constructed for him in the last 2 years. Examination show that the
dentures are perfect but he insists on making a new one and to know every step in
details. What is the type of this patient?
a- Bacteria initiated the caries
b- philosophical
c- cooperative
d- hysterical****
45

193. 64- Which one has a positive rake angle?


a- k file***
b- k reamer
c- profile
d- protaper
194. 65- During extraction of a mandibular second premolar; it slipped into the oral
cavity. how to retrieve it?
a- Allis forceps
b- Russian tissue forceps****
c- college pliers
d- Addison forceps
195. 66- The most important factor for the success of sealant is:
a- control salivary flow***
b- the sealant material
c- no tooth brushing before application
d- no tooth brushing after application
46

196. 67- What is the best description of successful application of oral hygiene
measures?
a- decrease probing depth
b- decrease plaque score***
c- decrease bleeding on probing
d- less number of carious teeth
197. 68- A small part of a periodontal curette wad broken and displaced in gingival
sulcus. how to retrieve it?
a- Schwartz periotriever***
b- tweezer
198. 69- Modification of complete denture by reinforcing acrylic resin by butadiene-
styrene rubber allows the patient to handle the denture more safely.
What do you call this type of dentures?
a- high strength complete denture
b- high impact complete denture***
c- immediate denture
d- transitional denture
199. 70- A 24 years old male pa ent presented by severe pain, halitosis, ulcerated
gingiva, easily bleeding gingiva. What is your diagnosis?
a- ANUP
b- ANUG***
c- lichen planus
d- neoplasm
200. 71- A young patient suffers from oral ulceration along with genital and ocular
symptoms:
a- Sjogren’s syndrome
b- Bechet syndrome***
c- Reiter syndrome
d- pemphigus vulgaris
201. 72- A 24 years old pregnant female at her first trimester is complaining from a
small lobulated mass at maxillary incisor area that bleeds easily.
a- acute periodontitis
b- acute herpetic gingivostomatitis
c- microcytic anemia
d- pyogenic granuloma***
47

202. 73- A photo like this one (but with no rests on 13


or 24) &A 55 years old female patient suffering
from pain and discomfort related to teeth number
13 and 24 and pain and bleeding related to the
area covered by this RPD. This problem started 1
month after delivery of this RPD. MW and rinsing
the area with saline improved her condition but the
problem became worse once she wears the
denture again. What would be the problem?
a- sensitivity to denture material
b- gum stripping design***= gum stripping means RPD without occlusal rest
c- no minor connectors
d- bad oral hygiene
203. 74- Where does the needle holder hold the needle during suturing?
a- at the end of the needle
b- at 1/3 the needle from the p
c- at 2/3 of the needle from the end
d- at 1/3 of the needle from the end***
204. 75- epinephrine containing local anesthesia is contraindicated in patients with:
a- hypothyroidism
b- hyperthyroidism****
c- hypotension
d- asthmatic bronchitis
205. 76- During which period of intrauterine life
the primary tooth bud is formed?
a- 4 weeks
b- 6 weeks
c- 8 weeks***
d- 14 weeks
48

206. 77- A pa ent complaining from dull pain in tooth number 46, clinical
examination shows occlusal caries with the following findings:
cold test: 10 seconds, hot test: 10 seconds, electric pulp test: feels pain a er 20
seconds, tooth number 47 was chosen as a control tooth with these findings:
cold test: 5 seconds, hot test: 5 seconds, electric pulp test: feels pain a er 8
second, how will you treat this tooth?
a- fluoride application
b- restoration
c- RCT***
d- extraction
207. 78- A 5 years old girl in an area with 0.2 Fluoride ppm, you will prescribe for her:
a- 0.25 ppm fluoride tablet daily
b- 0.5 ppm fluoride tablet daily***
c- 1 ppm fluoride tablet daily
d- none of the above
208. 79- What is the reason for most periodontal surgeries?
a- removing granulation tissue
b- remove all calculus
c- remove all plaque
d- expose diseased roots***
209. 80- How to differentiate
between acute and chronic
periapical abscess?
a- presence of fistula
b- presence of periapical
radiolucency
c- severity of pain
d- all of the above***
210. 81- A 34 years old male
patient is complaining from dull pain on bi ng on tooth number 35, Clinical and
radiographic examinations show poor RCT and recurrent caries under amalgam
filling, the patient feels pain only with biting which subsides by release of the bite.
What is your diagnosis?
a- periapical abscess
b- periodontal abscess
c- periodontitis
d- root fracture***
211. 82- Vitamin B12 deficiency is associated with:
a- microcytic anemia
b- macrocytic anemia
49

c- pernicious anemia***
d- aplastic anemia

212. Over exposure of nitrous oxide inhibits metabolism of vitamin:


A. Vit B12***
B. Vit B6
C. Vit A
D. Vit C
213. 83- Histopathological picture of -Ortho keratosis or para keratosis
Lichen Planus includes: -Acanthosis or atrophic
a- acantholysis -Infiltration of band of lymphocytes
b- acanthosis*** -Liquefactive degeneration of lymphocytes
-Civatte bodies (basal cells undergoing apoptosis)
c- thinning of granular layer
-Saw tooth profile of rete ridges
d- band like infiltration of phagocytes
214. 84- A 65 years old pa ent
complains of a painful mass at right side of the cheek (at buccal mucosal area). He
had the habit of tobacco chewing for long me but he stopped 2 years ago. This
lesion is covered by leukoplakic patches, and the patient has enlarged lymph
nodes. What is your diagnosis?
a- basal cell carcinoma
b- verrucous carcinoma***
c- leukoplakia
d- erythroplakia
215. Which of the following is most
important in determining the
prognosis for a tooth?
A. Probing pocket depth
B. Bleeding on probing
C. Clinical attachment level***
D. Level of alveolar bone
50

216. 1)Systolic 120 diastolic 80 Calculate the mean arterial pressure


2/3(diastole)+1/3(systole)=Map = 2/3 *80+1/3*120=93.3
218. 1. Concentra on of acidulated phosphate fluoride→ 1.23%
219. 2 Which fluoride can be applied professionally by the dentist
1. Act
2. Duraphat***
3. Gel n
4. Prevident
220. 3. When should be
the child first exposed
to tooth brushing→
When the primary teeth erupt
221. 4 How can u differen ate
between endodontic and non-
endodontic pathosis
1. Radiographic appearance
2. Radiographic loca on
3. Pulp sensibility tes ng***
222. 5. Long case about pt. -
paresthesia and moth-eaten
appearance →acute osteomyelitis
223. 6. About Schwartz Perioretriever
224. 7. About rubber dam, they said wat is the best stable position→I chose 4 points
contact between the beak and the tooth --- other options were not appropriate
225. 8, An OPG of a child showing mixed den on period and asked the age --- teeth
present were centrals and laterals all 1st molars, lower canine, patient having
diastema in anterior showing ugly duckling stage and upper canine about to erupt
1. 6 years
2. 10 years***
3. 15 years
4. 20 years
226. 9. Type of bone best for implant → type II
227. 10. Most important biochemical criteria for implant
A. Implant made of titanium alloy***
B. Coronal anti rotation
Rest don't remember
228. 18. An female pa ent came with complete denture came to the clinic with
complaint of old age appearance
A. Increase in leeway space
B decreased lower vertical height***
51

229. 11. How much mm of tooth structure should be present around post and core
A. 1mm***
B. 4mm
C. 6mm
N.B: Minimum of 1 mm of sound den n should be present, if tooth structure is
men oned then more than 2 mm should be there
230. 12. Acute consequence of vigorous flossing
A. Buccal gingival recession
B. Loss of interdental bone
C. Laceration of interdental papilla***
231. 13. Why is electric test not reliable in young adults →

232. 14. A child with mild pain related to


36, radiographically there is
radiopaque lines in rela on to 36 ...
Most probable diagnosis
A. Paget's disease
B. Fibrous dysplasia
C. Garre’s osteomyelitis ***
D. Central giant cell granuloma
233. 15. Child with bilateral swelling in
lower mandible region, on radiograph
multiple
radiolucency and
2nd molars are
displaced
anteriorly.
Diagnosis
A. Ameloblastoma
B. Fibrous
dysplasia
C. Cherubism****
52

234. Hypercementosis and ankylosis in which disease→ Paget’s


235. 17. Hemidesmosomes a ached to→ Lamina Lucida
236. 20. Pa ent with lower Kennedy class 1 and upper edentulous, what is the most
appropriate material of choice of teeth material for the posterior of upper
edentulous arch .... (specifically posterior they asked)
A. Porcelain
B. Metal
C. Acrylic***
237. 21. A case with a female pa ent missing 11.... Which of teeth should be used as
an abutment for best esthetic
A. 12
B. 21
C. 12 21***
D. 12 13 21
238. 23. Concentra on of chlorhexidine in mouth wash →0.12%
239. 24. About aluminum foil test→ test of ultrasonic cleaners
240. 25. Spedding principle→ used for the selection of stainless steel crown
241. 26. Use of sodium hypochlorite
A. As a mouth wash
B. To wash to extraction socket
C. Irrigant***
242. 27.Error in processing can be minimized or eliminated by
A clinical remounting
B. Laboratory remounting***
C adjusting the occlusal plane
243. 28. Which surface of distal root of mandibular 1st molar is more prone to
perforation while post preparation
A. Distal
B. Mesial***
C. Buccal
244. 29. Factor in hemophilia B→ factor 9
245. 30. Pa ent with trismus... A er giving IAN block → Vazirani-akinosi tech
246. 31. Pa ent given block for PSA.... S ll feels pain→buccal infiltration
247. 32. Pa ent having xerostomia
A. Abrasion
B. Abfraction
C. Erosion
D. Root caries***
248. 33. Pa ent with bulimia and saucer shaped lesion on the palatal of upper
anterior→ Erosion
53

249. 19. Which material is made by CADCAM / milling --- something like that
a. In ceram = strongest
b. Empress = esthetic
c. IPS empress
d. Cerac vita block or decor block***

250. 34. Pt with radiotherapy, how to extract the tooth in order to prevent
osteoradionecrosis → Extract under hyperbaric oxygen
251. 35. Branch of internal caro d artery →ophthalmic artery
252. 36. Manifesta on of Pericroni s
A. Vesicular eruption
B. Pain with hot and cold
C. Swollen suppurative
D. Acute inflamma on around erupted 3rd molar***
253. 37. Which of the following is surgically removed
A. Lower canine
B. Upper 3rd molar
C. Lower third molar***
54

254. 22. A case with renal transplanta on and pa ent has sore throat and burning
A. Leukoplakia
B. Pseudomembranous candidiasis***
C. Erythematous candidiasis

255. 38. Perfora ons in upper premolars due to


A. Large pulp chamber
B. Complex internal anatomy
C proximal root concavities***
55

256. 39. Oral surgeon referred an implant


patient for the crown placement, the
implant is in place since 3 months and
everything is perfect. The only problem is
that there is only 4mm ver cal space for
the crown placement.
a. Screw design ***
b. Refer to the oral surgeon.
c. Make it out of contact
d. Make esthetic abutment / resin
type
257. 40. The most destruc ve occlusal interference is
A. Centric
B. Working
C. Non-working***
D protrusive

258. 41.Pt underwent renal transplantation 3y ago he has white non-scrapable lesion
on the lateral side of the tongue appeared corrugated and has shaggy and frayed
what is diagnosis!!!
1-hyper plastic
candidates
2-Idopathic
leukoplakia
3-lichen plans
4-hairy leukoplakia***
259. 43. The main
advantage of
immersion technique
is
A. Prevent the
impression from
56

distortions
B. Helps all the surface of the impression to be disinfected***
260. 42. Deflec on of the par al denture increase
A square the length
B. Cube the length***
C. Square the width
D. Cube the width

261. 44. How long is the rinsing time of chlorhexidine in mouth wash to be effective
A. 15 s
B. 30 s****
C. 40 s
D. 50 s

262. 45. - white spot on tooth surface which partially or completely gets removed by
wetting → incipient caries
263. 46. What is prion protein→A. Resistant to heat
(https://en.wikipedia.org/wiki/Prion#Sterilization
57

264. 47. Tooth bud formed at which age of intrauterine life →8th week

265. 48. Not recommended for mouth


breathers →GIC
266. 49. Which is not the surface of impression
A. Impression surface
B. Polished***
C. Occlusal
D. Vestibular
267. 50. Carbohydrate which is easily metabolized by bacteria to produce caries
A. Glucose
B. Sucrose***
C. Maltose
D. Fructose

268. 51. Posi on of spreader while obturation- →1- 1.5 mm short of working length
269. 52. A pa ent pain related to tooth. Which is tender on percussion ... What is the
best management
A. Complete root canal of the distal root up to apex
B. Complete root canal treatment of distal root beyond the apex
C. Complete root canal treatment of the tooth with corrected working length***
58

270. 53. During ini a on and prolifera on what happens - Oligodontia

271. 54. Pa ent with amelogenesis imperfecta want to do ortho treatment →


Invisalign system
272. 55. Pa ent with routine dental checkup, there is found mixed radiolucent and
radiopaque related to the root of the teeth which is vital
A. Ameloblastoma
B. Radicular cyst
C. Periapical Cemental dysplasia***
D. Osteosarcoma
273. 56. Pa ent is given fluoride tablets ... Applied varnish ... Then came with side
effect. What shall we give→- sialagogue
274. 57. Pa ent with complete denture came to the clinic and said that the denture
moves a lot .... After u check and press on the distal end. Anterior end gets
dislodged ... What ttt
A. Reline
B. Rebase***
C. Replace
D. Do nothing
275. 58. Mother called den st and said that the child drank 50mg fluoride ... What
will the dentist advice
A. Check for the symptom and take to emergency room
B. Give acidic food and take to emergency room
59

C. Give nothing and take to emergency room


D. Give calcium and take to emergency room***
276. 59. Lu ng cement should be more in
A. Compression***
B. Tension
C. Shear
277. 60. The material for
FPD should have
A. Low strength
B. Low rigidity
C. High strength
D. High strength and high rigidity***
278. 61. Pa ent allergic to amid type LA .... Want to give prilocaine 4% 1:200000
lidocaine .... What is the color
A. Red
B. Blue
C. Yellow***(Black = prolicaine 4% plain)
D. Green
279. 62. A 26-year-old child came with complain of recurrent oral ulcers .... He gave
history of ocular and genital lesion
A. Brohn disease
B. Behçet's disease***
C. Lupus erythematosus
280. 63. What is the most imp property of MTA
A. It deals better than any other material***
B. Best tensile strength
C. It has better shear strength than others
281. Ibuprofen dosage for a child of 12 years old
A. 4 mg/kg q4h
B. 4 mg/kg q6h
C. 6mg/kg q4h
D. 6 mg/kg q6h***(5-10mg/kg q6-8h)
282. 65. Which is not present in saliva‫؟؟؟؟؟؟؟؟؟؟؟‬
A. Lactoferrin
B. Antibodies
C. Potassium
D. Proteolytic enzyme
283. 1_image of class 1 n they ask what is defect in this design a er 3 years pa ent
feel mobility if grade 2 in second premolar which is abutment
a. inappropriate indirect retention
b. poor major connector
60

c. inappropriate rest***
284. 2. Image with recession of gums in anterior area pa ent with high
hypersensitivity what will be ur treatment plan??
a. Application of fluoride varnish by professional dentist ***
b. fluoride application in custom tray in clinic
c. application of FL in tray at home
285. Image of compound odontoma
286. Treatment of ossifying fibroma

287. Treatment of Mucocoel→ excision


288. Time period of splinting tooth after avulsion → 2wks

289. White lesion on tooth go away when u wet the tooth surface→ incipient caries
290. What is the thing in rpd which prevent tissue word movement of denture??
a. Rest****
b. indirect retainer
61

c. major connector

291. A pa ent come with (8y old) notched incisors n mulberry molars → Congenital
syphilis
292. What is the amount of FL in water, 0.8 i chose→1 mg/ml.
293. A tooth with large amalgam restoration n patient feel pain throbbing n apical
abscess n u can’t touch tooth??
a. Acute pulpitis with chronic periodontitis
b. Chronic pulpitis with acute periodontist
c. Irreversible pulpitis with acute abscess***
294. Property of sealer??
62

295. The preparation of inlay is


a. larger then amalgam ***
b. smaller than amalgam
c. Same
296. Definition of hardness

297. Cbct property??


a. Good tmj visualization
b. grater amount of radiation as normal radiographic technique ****
c. more accurate
298. Gingival pigmentation green → bismuth
63

299. Sjogren’s syndrome patient

300. Dentigerous cyst with long patient history


301. An image of pa ent with upper 11 absent n lateral with carious lesion from
where we start our treatment
a. scaling
b. Restoration of lateral ***
c. denture
302. A v confusing pic of fpd with pier abutment n the tooth is upper 5 n ge ng
reten on from 7 n 3 .... n pic showing stresses where should be the main
retention??? I did not understand that
303. A scenario with values for tooth n diagnose from values n that was irreversible
pulpitis bcoz the pain on hot was prolong time
304. What is the radiographic tech being appropriate while planning for implant?
a. occlusal
b. periapical
c. panoramic **** (CBCT 1st, MRI 2nd)
d. CT
e. MRI
305. A child come with dog bite how can u treat→Convert punch to laceration than
suture
306. A patient came with caries in pit n fissure n only in enamel treatment??
a. sealant
b. conservative restoration ****
64

c. restoration

307. Schwartz Perioretriever

308. Angel of perio while working 40 to 90


309. Epithelium of maxillary sinus→ Ciliated Pseudostratified columnar epithelium
310. Opening of maxillary sinus →in middle meatus
311. A patient on dialysis treatment time →after one day
312. A patient with white lesion ... when swab red bleeding surface →its candida
albicans
65

313. Antibiotic Treatment of lefort 1→ Reduction and fixation

314. for aggressive periodontist→ its tetracycline


315. What is the cement good for pulp →its poly carboxylate bcoz it does not irritate
the pulp
316. Pink spot tooth→internal resorption
317. GG burs use for →Coronal part of root preparation
318. Which is contraindicated in soft carious removal??
a. round bur with slow speed
b. round bur with high speed with low........****
c. with spoon excavator

319. Which is component of rpd which


is responsible for stress distribution loads→ Major connector= bear load
Minor Connector = Distribute load
320. Occlusal relation for mounting ....
a. centric occlusion ***
b. Centric relation
c. eccentric relation
321. Dynamics of caries→ Demineralization with remenrlization
66

322. Hysterical child management??→poose board/ strap i think so


323. Role of epinephrine in Apicectomy→Prolong duration of anesthesia
324. Role of vasoconstrictor in L.A →decrease toxicity
325. Apex of file 20 is→ 0.2mm
326. 5-year-old child with missing upper lateral→ No ttt
327. Purpose of pouch before sterilization →
preserve sterility of inst. To prevent rusting
I’m not sure for this
328. Problem in calcification of tooth at which
stage it occur
a. initiation
b. Histodifferentiation
c. Morphodifferentiation
329. Image of one pier abutment with grade 2
mobility n its upper 5 with amalgam
restoration what should be the treatment
plan
a. fpd
b. rpd
c. rpd with overdenture abutment***
d. ext. of 5
330. A person with bad oral hygiene & multiple caries → oral hygiene instructions
then restoration
67

331. One question about condensing osteitis

332. Amount of FL dose depends on


a. age ***
b. Temperature
c. water FL***
333. Improper periapical film placement cause a. shortening of roots b. elongation of
roots c. overlapping??????
334. Active appliance → posterior bite block as in files
335. Percentage of fracture teeth in maxilla → 75
336. Patient with trismus→Vazarani Akinosi tech
337. A tooth with fracture cusp dentine involve what is the status of pulp in this case
a. hypersensitivity
b. reversible***
c. irreversible
68

338. Function of water stream device


a. plaque removal
b. prevention of plaque
c. dilution of bacterial toxin***
339. Before sealant application→ polish with paste, apply etchant& then rinse and
dry then apply sealant
340. What is imp in sealant application→isolation
341. Definition of fossa

342. On tooth surface bacteria do


a. quorum sensing****
b. quorum signaling

343. What is present in alginate which imp for its setting→Calcium sulfate
69

344. What is the margin for porcelain fused metal crown? → Shoulder in facial
surface(porcelain) Chamfer on lingual (metal)
345. What is the function of flux in soldering → flow of current

346. Treatment of mahler class recession ??→ CT gra under par al split for class 2
347. Conc of home NaOCL →5.25
348. Upper two central n lateral fracture n bone moving as a whole on palpation
what u see on radiograph????
a. a wide gap ***
b. lamina dura separated from root apex
349. In serum of patient u
find antibody HBAg
against HBv what is it
a. immunity
b. chronic inf
c. acute inf.
70

350. At what age visit orthodontist→7 to 9 when upper permanent incisors erupt**

351. A tooth with no mobility but laterally luxate age of pa ent 9 years→ Permanent
splint., Primary exo
352. 9 years old child you note white lines which are more concentric in cervical than
incisal ... identify
a-enamel lamella Retzius stria in enamel cervical.
b-von ebner straie Von Ebner in dentine.
c-enamel tuft Lamella vertical line on tooth crown.
d-Wickham straie Answer here E choice Retzius stria
353. 1. Fordyce’s spot on lips
are composed of which type
of glands?
a. Sebaceous***
b. Serous
c. Sweat
d. Mucous
354. 2. Irreversible pulpi s
a. No pain on cold
b. Sharp pain on hot ***
c. Mild pain with cold
d. Severe pain with cold
355. 3. Trauma to tooth
causing pulp exposure, no caries, no previous restoration.pt. feels sharp non-
lingering pain with cold. Tooth is sensitive to touch. answer?
a. Asymp irreversible pulpitis with Asymp apical periodontitis
b. Asymp reversible pulpitis with symp apical periodontitis *****
c. Asymp irreversible pulpitis with symptomatic apical periodontitis
356. 4. Trauma to tooth. dentine exposed. pain on cold pulp response?
a. normal and uninflamed pulp
b. rev inflamed****
c. irrev. inflamed
d. Necrosed
71

357. 5.Incipient or recurrent caries can be detected before they are visible on a
radiograph by:
a. Visible light
b. Ultrasonic light
c. Fiber optic trans-illumination
d. Digital fire optic trans-illumination***DOFTI
358. 6. 9 years old child in area with 0.1 ppm water fluoridation... what to give
a. 1 mg****
b. 0.5 mg
c. 0.25 mg

359. 7. Tip of tongue drains into →submental lymph nodes


360. 8. Distance between cephalogram and patient.
5 feet ***
6
7
8
361. 9. Expiry of amalgam restora on
20 years ***
5 years
7 years
362. 10. OKC radiograph scenario, Picture
something like this but instead of one big
radiolucency there were two teeth were
displaced in similar manner and pts complain
was moving teeth. his father also had removal
of lesions in the past
72

363. 11. Desired resorp on in


ortho tooth movement
a. Hyalinization ***
b. Undermine resorption
c. Alveolar resorption
364. 12. Resorp on of root at periapical area depends on
a. Magnitude of force****
b. Duration of force

365. 13. Lateral periodontal cyst scenario


366. 14. Pt reported to dental clinic. Den st concluded that pt. is at high risk of
caries. Pt prescribed dentifrices and asked to avoid sugary foods. 2 days’ later
patient presented with a side effect. what to do →Prescribe a sialagogue or
salivary enchoregment
367. 15. Right D missing. Right permanent molar about to extract. Le D and le 1
molar present. Space maintainer for right side?
a. Lingual arch
b. Band and loop***
c. Reverse band and loop
368. 16. Osteomyeli s characteris c feature? →Sequestration
369. 17. Rela on between tooth surface and instrument →Adaptation
370. 19. Pt reported with bluish black swelling at site of unerupted 46. Radiograph
reveal lesion is confined to soft tissue only. treatment
a. Lesion removal and trac on of 46
b. Cauteriza on of lesion and close follow up of 46 to extract
c. Simple incision on roof of lesion and wait for 46 to erupt***
73

371. 18. Caries affec ng


children
a. Rampant
b. Early childhood***
74

372. 20. Implant abutment removed. picture was there taking impression and
something was attached in place of implant. i choose implant analogue

373. 21. Porosity on cast a er taking imp with polyvinyl siloxane. →release of
hydrogen gas
374. 22. Anesthesia insufficiently achieved by inf alv nerve block for man 1st molar
Bcz patient feel pains in mesial portion of tooth. dentist gave additional
anesthesia. to anesthetize which nerve
a. lingual
b. mylohyoid
c. auriculotemporal
d. mental***
375. 23. Indirect reten on not needed in which class.
1
2
3***
4
376. 24. Water irriga on device to →dilute bacterial toxin.
377. 25. Main func on of floss→To remove interproximal plaque
378. 26. Calculus consists of → mineralized plaque
379. 27. prevalence of disease in a group now is being compared with prevalence of
disease in a group 6 years ago. study?
a. Cross sec
b. Cohort***
c. Case control
380. 28. Forcep for
epulis fissuratum
a. Allis***
b. Addison
c. Stillis
75

381. 29. Pain a er extrac on


a. Persis ng over 3 days is alveolar osteitis (= dry socket) ***
b. Peak a er 3 days
c. Resolve in 5 days

382. 30. Hard caries removed by (incomplete question)


High speed***
Slow speed
Spoon excavator

383. 31. Guiding planes are


Occlusal
Buccal = maxillary
Lingual= mandibular
Mesial
384. 32. Retreatment is easy in →Weeping canal
76

385. 33. Blow to right side. Symptoms were given for zygomaticomaxillary fracture

386. 34. Advantage of pouch during steriliza on→To prevent rusting


387. 35. An bio c inhibi ng bacterial cell synthesis →penicillin
388. 36. Which instrument has posi ve rake angle
a. K file (its k confirmed) ****
b. Protaper
c. Profile
d. K reamer
389. 37. Best material for root caries
a. Resin mod gic***
b. Compomer
c. Composite
d. Amalgam
390. 38. Primary stress bearing area of mand
Buccal shelf area***
Residual ridge
Retromolar pad
391. 39. Pa ent with upper complete denture complaining from poor reten on and
after examination we found bubbles in posterior area of upper denture, why?
A. Under postdamming***
B. Over extension
C. Under extension
D. Over post damming
392. 40. What is used to record jaw rela on
ZnO
Base plate wax***
77

393. 41. Fluoridated toothpaste for 3 years child is:


a. Recommended. ***
b. Not recommended.
c. Limited
d. Toxic
394. 42. Maryland rpd for
Post teeth
Translucency of ant teeth***
395. 43. Phone cs affected by
a.Short max teeth
b.Placing max anterior teeth above occlusal plane and at distance from lower lip***
c. Improper positioning of premolars and molars
396. 44. Reduc on of cusp for complex amalgam rest
a. 1 to 1.5 mm
b. 1.5 to 2***
c. 2 to 3
397. 45. leads to separa on of tooth
a. During operative procedure
b. Vertical root fracture***
398. 46. Injury to which nerve for gag reflex→Glossopharyngeal
399. 47. Caries most common in →Pits and fissures
400. 48. Digital fiber optic transillumination is for
a. All surfaces**
b. Proximal
c. Pits and
fissure
d. Smooth
surface

401. 49. Intra pulpal arterial blood pressure→10mm of hg


78

402. 50. A er class V GI restora on removal of a thin flush of GI is done by:


a.Scaler or knife immediately. *
b.Finishing stone immediately.
c. Scale or knife later.
d.Finishing stone later. *
1 a + b.
2 a + d.***
3a+c
4d+c
403. 51. Most common material used for apexogensis→MTA
404. 52. Which material used in one-visit Pulpectomy:
a. MTA ***
b. Ca (OH) 2 + CMCP
C. Formocresol
D. Zn oxide
405. 53. MTA is advantageous over other materials Bcz of →Sealing properties
406. 54. Etching for porcelain
a. 4 to 10 % hydrofluoric acid****(9.5%hydrofluoric acid)
b. 34 % hydrofluoric acid
407. 55. Bone death from heat during implant...?
1-42 c for 30seconds
2-42 c for 1 minute
3-47 c for 30 seconds
4-47 c for 1 minute***
408. 56. Pa ent with radiopacity in the periapical area of 1st mandibular molar with
a wide carious lesion and a bad periodontal condition is:
a. Condensing osteitis***
b. Cemental dysplasia.
c. Periapical granuloma.
d. Hypercementosis
409. 57. The access opening in lower incisor:
a. Round.
b. Oval. ***
c. Triangular.
410. 58. Which test is not used for pulp vitality →Percussion
411. 59. Subgingival instrument
Chisel
Hatchet
Sickle
Gracey curette***
79

412. 60. Does not interfere with healing


Antibiotic *
Long term steroids
Infection
413. 61. Final needle aspira on needle gauge →19 gauge
414. 62. Child drank 2/3rd of mouthwash bottle contains FL immediate treatment
→Give milk and calcium gluconate
415. 63. Punching holes in rubber dam too closely will →Inability to place rubber
dam interdentally
416. 64. PDL ligament reparative cells are → fibroblasts
417. 65. Related to den nogenesis
a. Dentin is formed at root first
b. Odontoblasts becomes cuboidal in late stage ***
c. Dentine matrix is involved
418. 66. NaOCL in root canal →Kills microorganisms
419. 67. Nerve supplying palatal surface of upper ant teeth →Nasopalatine
420. 68. Scenario for denture stoma s
421. 69. Scenario for pulpal necrosis
422. 70. Class 2, overjet of 5 mm, profile? →Convex

423. 71. most characteristic difference between bone rarefaction and radicular cyst
is →vitality
424. 72. Stainless steel crown selected for child, during try in the crown rotates freely
on the tooth.
a. Add a thick layer of cement before cementation
b. One size smaller than this crown ***
c. Small size crown
80

425. 73. image for child intraorally has ant swelling palatally and buccally including ant
teeth, histopathology the lesion has multiple giant cells? →giant cell granuloma
426. Mercury ingestion during placing amalgam filling
a. 5 _4
b. 1_2
c. 45*** (1-50 mg per cubic meters of air the vapor of mercury.)
d. 95
427. Pat has normal centric occlusion and interference in eccentric we should do
grinding to:
a. incisal edge of max ant
b. incisal edge of mand ant***
c. lingual inclination of max ant
d. Lingual inclination of mandibular ant
428. Tooth with yellow discolouration the cause is
a. Internal resorption
b. Necrosis
c. Obliteration of canal ***
d. Hemoglobin in canal
429. Area not important in post damming
a. Vibrating line
b. Hamular notch
c. Retromolar pad***
d. Palatine fovea
430. Treatment of internal resorption
a. Condensed with amalgam
b. Remove remnant pulp tissue ***
c. Prepares apical to the resorption area
d. Silver cone w sealer to the resorption area
431. Purpose of immediate denture
a. To improve appearance ***
b. To improve function and mastication
c. To decrease resorption of the alveolar bone
d. To maintain the facial height
432. Patient after taking local anesthesia he starts to have nausea and he becomes
warm with blood pressure 100/75 i think he was trea ng from osteoarthri s what
is the diagnosis
a. Hyperthyroidism
b. Hyperglycemia
c. Adrenal insufficiency ***
d. Adrenal crisis (it is the best answer if present in the options)
81

433. The main component of plaque


a. Bacteria***
b. Food
c. Saliva
434. Pat with RPD have fracture in the rest. This is because
a. Rest set reduction is made more than central fossa
b. Rest is not well adapted*
435. Q about tmj diagnostic x-ray except
a. Arthrography
b. Chondro
c. Transcranial***
436. Local anesthetic solutions need which media to work or something like that
a. Lipid ionic
b. Lipid nonionic***
c. Water ionic
d. Water non-ionic
437. Pt came with trauma one hour ago in his tooth with moist pulpal exposure (=
contaminated) what is your treatment
a. Direct pulp capping
b. Indirect pulp capping
c. Rct ****
d. Extraction
438. In which area of the tooth we put pin in amalgam restoration to increase
retention
In most bulk area of dentine ***
I don't remember the others
439. How can we differentiate between periodontal and apical abscess
X-ray
Vitality test ***
482. intrapulpal pressure
10mmHg
483. after orthodontic
treatment which surface
is most demineralized,
a. behind brackets
b. around brackets**
82

484. 58 years old man is going to undergo surgery for carcinoma, 80kg, which is the
maximum amount of lidocaine 2% with epinephrine 1:100000 is allowed (
→Lido w epi. 7mg per kg, But without epi 4.5 mg per kg
-This q with epi. So 80kg X 7=560mg
-To calculate how many carpule in one visit = 560 /36(= 20mg/ml X 1.8 ml) =15.5=16
485. 6 years old kid comes wicked skin rare hair, 2 peg shaped teeth only, which

syndrome? →Ectodermal dysplasia


486. skin area above mandibular angle, which nerve? → great auricular n
83

487. image + 5 years old boy comes, small stature, had mul ple fracture of bones
before, image of hypodense dentin→ Osteogenesis imperfecta
488. lactating mothers needs AB which is the first choice:
a. ampicillin***
b. metronidazole
c. tetracycline
489. image of blue lesion swollen in the extremity of the tongue:
a. hemangioma***
b. lipoma
c. fibroma
d. Neurofibroma
490. child 12 years old consults for brown deposi on in permanent molars but
primary molars are not fluorotic why?
a. primary dentition calcifies in less time*
b. child uses high fluoride toothpaste after eruption of molars
c. fluoride is filtrated by mother's placenta
491. tooth 46 pain at percussion, cold nega ve, hot nega ve, electric test negative,
what is the diagnosis
a. reversible pulpitis
b. symptomatic irreversible pulpitis
c. asymptomatic irreversible pulpitis
d. necrosis***
492. tooth 36 pain that lasts on cold and hot, electric test nega ve, percussion
negative, what treatment:
a. RCT***
b. Crown
c. restoration
d. extraction
493. how to evaluate that a periradicular periodontitis is healed?
a. lesion is disappearing
b. no more lesion
c. no more symptoms ***
d. no more sinus tract
494. which instrument is location specific in periodontal treatment:
a. sickle
b. hoe
c. gracey ****
495. how to evaluate tissue healing after perio treatment:
a. pocket decreasing on probing***
b. hypertrophy decrease
c. decrease of plaque index
84

d. no more bleeding
496. angle of instrument in perio with dental surface→Angula on 70-80
497. which nerve for mesiobuccal root of first maxillary molar→Middle superior
alveolar
498. which root has the most risk to penetrate in sinus after fracture→Palatal root of
max first molar
499. trauma intrusion of 2 central permanent incisors with open apices:
a. apexogensis
b. surgical extrusion
c. orthodontic extrusion
d. observation***
500. in children which are the teeth that are most affected or first affected?

501. old man with multiple recessions and


cervical caries which affect roots, which material for restoration:
a. amalgam
b. resin
c. glass ionomer
d. reinforced glass ionomer****
85

502. intraligamentary anesthesia in tooth 46 (nerve block and glow gates not
efficient):
a. blood circulation in tooth is stopped for
30 min
b. not stopped,
c. increased,
d. decreased***
503. a case of eruption hematoma
504. one radiolucent image 2 cm of diameter,
extends 6 mm below molar and premolar roots, and 10 mm towards inferior limit of
mandible, vital teeth: traumatic cyst, Dentigerous cyst, odontoma, (don't remember
4th choice) →Stafne cyst= Bone cyst
505. trismus and no possibility of nerve block:
a. incisive block
b. gowgate
c. Akinosi***
d. Supraperiosteal
506. anesthesia buccally first maxillary molar:
a. submucosal
b. Supraperiosteal***
c. subperiosteal
507. percentage of copper to eliminate gamma2 phase→13%
508. amalgam for retrograde obturation:
a. free copper
b. free mercury
c. high copper***
d. with AB
509. function of pituitary gland:
a. calcium secretion
b. growth factors***
510. how to guarantee amalgam retention
a. convergence of walls***
b. divergence of walls
511. dentine adhesion 56 years old woman with epilep c problem, renal
calcifications and other stuff has pain in mandibular molar area unilateral + image
(there's a horizontal bone resorption and maybe hypercementosis):
a. Hyperthyroidism
b. Hyperparathyroidism***
c. Hypothyroidism
86

d. hypoparathyroidism

512. most favorable crest:


a. square
b. undercut
c. ovoid***
d. taper
87

513. how much taper for preparation for crowns→6


514. what is hybrid layer in dentin bonding systems?

515. Languer's surgery for CL II recession


a. total flap under total flap
b. total flap under split flap
c. graft under total flap
d. graft under split flap****
516. contraindication of gingivectomy

517. which type of articulators is nonadjustable:


a. Cl I = nonadjustable
b. II= nonadjustable
c. III=semi-adjustable
d. IV=fully-adjustable
88

518. which is the most


flexible instrument for
endo:
a. k file
b. reamer***
c. hedstorm...
519. most characteristic
of irreversible pulpitis:
a. pain on cold,
b. pain on cold that
lingers***
c. pain in
percussion
520. custom tray for final
impression in FPD:
a. strength
b. flexibility
c. rigidity***
d. thickness

521. ideal water fluoridation →1ppm


89

522. when to plan the closure of the lip in a cleft lip case

523. In revascularization case ... what length time leave triple antibiotic+ what is L.A
used +what material used for blood clot
A-3 week,3%mepivacaine without epinephrine, MTA***
B-2 week, 2% lidocaine with 1:100000 epinephrine, GIC
C-2 week,2%ar caine with epinephrine, composite
N.B: Example of a Revascularization Protocol
 Based on current research, several considerations can be reviewed when
considering a protocol for revascularization treatment.
 Indications: The non-vital incompletely developed permanent tooth with open apex.
 Although the ultimate goal of this approach is to develop a tissue engineering–
based method of pulpal regeneration in the fully developed permanent tooth,
 The current revascularization protocols have not been developed or evaluated for
these more challenging cases.
 2 visit protocol
A. At the first appointment (examination & disinfecting visit)
1. collecting clinical information and establishing pulpal and periradicular
diagnoses
2. the treatment alternatives, risks, and potential benefits should be described to
the patient and guardian
3. informed consent, include
a. the number of appointments (at least two),
b. the possible adverse effects (primarily potential minocycline staining of the
crown)
i. Clinical staining of the crown and any root structure above the gingival
margin appears to be due to the presence of minocycline.
90

ii. This can be minimized by using a delivery system that restricts the drug
below the cementoenamel junction (CEJ).
iii. When crown staining does occur, it can often be reduced or eliminated by
a walking bleach method with sodium perborate. likely alternative
treatments would
c. the potential lack of response to treatment and
d. alternative treatments, including MTA apexification, no treatment, or
extraction.
e. possible posttreatment symptoms
4. the tooth is anesthetized, isolated, and accessed.
5. Minimal instrumentation should be accomplished,
6. use of a small file to “scout” the root canal system and determine working
length is important.
a. If sensation is experienced within the canal system, this may suggest that
some residual vital pulp tissue remains
7. The root canal system is copiously and slowly irrigated with
a. 20 ml of NaOCl followed by
b. 20 ml of 0.12% to 2% chlorhexidine (CHX).
c. Irrigation criteria
i. place the needle into the apical third because of canal disinfection relies
considerably on chemical irrigants
ii. irrigate using needles with a closed end and side-port vents (e.g., Max-I-
Probe needles),
iii. together with a slow rate of infusion, to help to reduce any irrigants
passing through the open apex.
8. The root canal system is then dried with sterile paper points, and
9. the antimicrobial medicament is delivered into the root canal space.
a. The best available evidence would support the use of either a triple
antibiotic paste or Ca(OH)2
i. Both medicaments have been shown to be effective
ii. The triple antibiotic paste has the advantage of being a very effective
antibiotic combination against odontogenic microorganisms but its
disadvantage is minocycline staining of the crown.
iii. Ca(OH) has the advantage of being widely available and is a
commonly used medicament, but it may be cytotoxic to stem cells.
10.After antimicrobial medicament is placed, the tooth is then sealed with a sterile
sponge and a temporary filling (e.g., Cavit), and
11. the pa ent is discharged for 3 to 4 weeks.
91

B. At the second appointment (revascularization starting visit)


1. the patient is evaluated for resolution of any signs or symptoms of an acute
infection (e.g., swelling, sinus tract pain, etc.) that may have been present at the
first appointment.
2. The antimicrobial treatment is repeated if resolution has not occurred.
3. In most cases, the acute signs and symptoms have resolved. Since revascularization-
induced bleeding will be evoked at this appointment,
a. L.A.
i. the tooth should NOT be anesthetized with a local anesthetic containing a
vasoconstrictor.
ii. Instead, 3% Mepivacaine can be used, which will facilitate the ability to trigger
bleeding into the root canal system.
4. Steps
 Following isolation and reestablishment of coronal access,
 the tooth should be copiously and slowly irrigated with 20 ml NaOCl, possibly
together with gentle agitation with a small hand file to remove the antimicrobial
medicament.
 After drying the canal system with sterile paper points, a file is placed a few mm
beyond the apical foramen, and the apical tissue is lacerated with bleeding up to
3 mm from the CEJ.
 A small piece of Colla-Plug may be inserted into the root canal system to serve as
a resorbable matrix to restrict the positioning of the MTA.
 About 3 mm of MTA is then placed, followed by a restoration.
A 12-18-month recall should be considered as the earliest time point to conduct
the clinical examination and evaluate continued radiographic improvement in
root development.
92

524. definition of incipient caries

525. the reason behind


maxillary premolar
extraction in class II with
long face→ we shouldn't
actually extract in this case)

526. 28 years old female is advised to brush her teeth forcefully to prevent calculus
why? →disorganize plaque
527. teacher is controlling the work (RPD) of third year student, he noted that he
forgot an element which distributes forces to teeth and prevents ward or (wart)
on the mucosa, → occlusal rest.
528. in which phase enamel hypoplasia occurs:
a. initiation
b. proliferation
c. differentiation
93

d. apposition***

529. Enamel is derived from: -


A. Ectoderm. ****
B. Mesoderm.
C. Endoderm.
D. Undifferentiated mesenchymal
cell.
530. Mucogingival junction separates:
a. Gingiva and alveolar mucosa. **
b. Junctional epithelium and
Sulcular epithelium.
c. Marginal gingiva and
attached gingiva
d. Periosteum and alveolar
mucosa.
531. Body retain 90% of fluoride in:
A. Kidney
B. Blood plasma
C. Calcified tissue***
94

532. what is flash sterilization....

533. pt. with constricted palate and open bite what the case →Thumb sucking or
Apert syndrome

534. child comes to clinic with notched central incisors with history of congenital
syphilis ... which stage the teeth is affected
a. Morphodifferentiation***
b. Histodifferentiation
c. apposition
d. proliferation
535. 47 Black woman has cheek pigmenta on from 2 months it increases in
pigmentation during the two months, histopathological acanthosis, hyper
keratosis,
A melanoma
B. pigmented nevus
C. melanoacathoma**
95

536. Minimum incisal liability


a. 3
b. 4
c. 6***
d. 8

537. 47 Black woman has cheek pigmenta on from 2 months it increases in


pigmentation during the two months, histopathological acanthosis, hyper
keratosis,
A melanoma
B. pigmented nevus
C. melanoacathoma**
538. The high risk disease for dental care workers from blood born
A. hepatitis b***
B. hepatitis c
C. Immuno deficiency disease
D.ITI
539. Same question but say dental care workers are more liable to which infection
A. hepatitis b
B. hepatitis c
C. HIV
D. influenza***
96

540. Tooth hot test pain at10 sec, Cold test pain at 15, Electric 10
Reference tooth, 5,5, Electric a er 25
A. healthy pulp
B. rev pulpitis
C. irrev. Pulpitis****
541. Same question but different options about treatment
A. restoration
B direct pulp capping
C. rct*********
542. Pregnant woman upper anterior swelling easily bleeds →Pyogenic granuloma
543. Recovered Pericronitis
A. Pepto streptococcus +fusobacterium***
B. Bacteroides +......
C. staphylococcus aureus+....

544. First visit to orthodontist


A. When all permanent erupt
B. when spacing
C. crowding***
545. Second year student found missed important components of removable partial
denture that prevent tissue ward movement and moderately distribute forces
across the supporting tissues→ Occlusal rest
546. Image rpd sever soreness around 13.21.23 and moderate inflamma on under
distal extended edentulous area
A.no rest seats gum stripper***
B.no stress breaking
C.no tissue stop
D. nonsufficient tissue relief
547. Pericoronitis
A. Acute inflammation around erupted tooth**
B. swollen suppurative
C. pain on cold and hot
97

548. Cross section of k - reamer


A. triangular***
B. square
C. circular

549. Rubber dam clamp number for unerupted molar


A.14A***
B.W4A...
98

550. What's true for rubber dam


A. Use elastic for badly decayed to ensures salivary isolation
B. 4 contact point on tooth and peak***
C. use elastic clamp for....
551. Cement dangerous to pulp
A. zinc phosphate***
B.ca oh
552. A root seldom has 2 pulp canals
A. mesiobuccal root of upper molar
B. mesiobuccal of lower molar***
C. distobuccal root of upper molar
553. Dental materials are classified as
A. ceramic, polymers and composite
B. ceramic, polymers and alginate.
C. ceramic, polymers. Cement
D. ceramics, metals, polymers,
composites***
554. Gates glade used for
→coronal part preparation
555. GG#1 of p diameter→50
Color code of prilocaine with epinephrine
A. blue
B. green
C. yellow****
556. During 3rd molar extrac on tuberosity become mobile →splint tooth and
extract a er 6 weeks ‫دي اﻻﺟﺎﺑﮫ اﻟﺻﺢ‬
557. Length of anesthetic needle inserted in inferior alveolar nerve block→two third
of long needle ‫دي اﻻﺟﺎﺑﮫ‬
558. recession class 2 treatment →connective tissue graft with split flap ‫دي اﻻﺟﺎﺑﮫ‬
559. Barbed Brouch used for
A. remove cotton from the canal***
B. coronal orifices enlargement
C. shaping of narrow canal
99

560. Pterygoid plate and lateral Pterygoid muscle comes of which bone
A. temporal
B. sphenoid***
561. Healthy patient when You give inf alveolar n block he warmed and loss
conscious
A. adrenal crisis
B. vasovagal attack***
562. The most cause for suffocation is patient swallowed →tongue
563. To prevent hypoxia for nitrous oxide patient→give 100% oxygen 0% nitrogen
564. 6 years’ child made a bitewing from6 months no caries now he is following up
what survey required
A.2 bitewing + panorama
B.2bitwing+ 2 periapical***
C.2 bitewing+ CT
D.no extra films required
565. Patient has no caries fluoridated water good oral hygiene follow up
A.3 months
B.9-12m***
C.12-18m
566. Trea ng over jet of 7 mm decrease
A.12
B.14
C....

567. Female patient with maxillary anterior multiple radiolucencies and radiopaque
related to vital
A. central giant cell granuloma***
B. periapical cyst
C. Cemental dysplasia
100

568. Dose of trigeminal neuralgia drug in daily divided dose


A.1000
With the titration of dose of carbamazepine, raising up to 1200mg
B.1200***
daily in three equal divided doses added with amitriptyline, we
569. Fluoride
content in could give relief of pain to patient.
ppm in
counter dentifrices
A.12000
B1000***
570. Analgesic to renal disease patient
450 acetaminophen ****
Aspirin
Brofen
571. For caries removal
A. acid fuchin
B. basic fuchin
C. propylene glycol***
572. Swelling anterior maxilla most common with
Upper molars
Lateral incisors****palatally
Central incisors
573. Two vital upper central incisors a radiolucency between them
A. incisive foramen***
B. nasolacrimal gland
C. abscess
574. Minimal Remaining tooth structure around post and core
0.5
1****
1.5
2
575. 1/during impression op RPD w removing it adhere and attached between teeth
what the problem→ improper mixing
576. 2/pa ent w hemophilia B need factor → factor 9
577. 3/ implants always made from. → titanium
578. 4/ radiograph of TMJ perfora on. → arthrography
579. 5/ when to use proxy brush. → type 2 embrasure & grade III furcation
580. 6/ image of lingual tori
101

581. 7/ microorganism of caries

582. 8/ anatomical structure between lower premolar


102

583. 9/ Hemidesmosome of basal cell epith. Attached to →lamina lucida

584. 10/child 6 years has abnormal den n & pulp


585. 11/ ques on about hyperalgesia
586. 12/ displacement of disc& condyle
587. 1. Cholinergic drugs and an -cholinergic drugs

588. 2. Ameloblastoma
589. 3. Odontogenic cyst
103

590. 4.amalgam gamma 2 how many percent of copper →low copper amalgam

591. 5. File 20tip = 2mm


592. 6. Schwartz periotriever → removal of broken perio instrument in the gums
593. 7.celluli s
594. 8. Lefort
595. 9. What age start ortho treatment→7yrs old
596. 10. Best anesthesia for pregnant

597. 11. Radicular cyst


598. 12. Mucocele→excision
599. 13.granuloma
600. 14.white lesion →incipient caries
601. 15.best irriga ng solu on for endo - sodium hypochloride
602. 16. About surfactant in endo
603. 17.ranula.
604. 18. Compa ble cement for composite- Caoh
605. 19.cement for Buccal groove-gic
606. 20.fluoride varnish what is the brand
2017

300-SLE QUESTIONS

NEW EDUTION
DR.AMR HATEM ABDULKAWI

[COMPANY NAME] | [Company address]


‫ﺑسم ﷲ الرحمن الرحيم‬

‫تم ﺑحمد ﷲ وتوفيقه تجميع اسئله ملف وتعديلها ولكن‬


‫هناك ‪:‬‬
‫‪-‬مﻼحظات مشار عليها ﺑاللون اﻻصفر منقوله من‬
‫الملف القديم‬
‫‪-‬وهناك اسئله غير مجاب عليها ﺑعضها جاوﺑت عليها‬
‫حسب معلوماتي مشار عليها ﺑاللون اﻻصفر‪.‬‬
‫‪-‬اﻻسئله اجاﺑتها تحت السؤال ملونه ﺑاللون‬
‫اﻻحمر‬

‫وﺑالتوفيق للجميع‬
Q1- innervation for lower ant. Teeth? Incisve nerve

Q2- which motor nerve causes gagging ? Vagus nerve

Q3- the concentration of Flouride in tooth-


1000 PPM- 1ml.
bast?

Q4- the histological feature ( rete ridges) Oral lichen planus


appear in which disease ?

Q5- Fluoride concentration in APF ? 1.23%

Q6- pt all his teeth are mobile and need to


be extracted, in which order you extract 87452163 (maxillary 1st ,posterior molar
them ? except 6 then ant. Except 3 then 6 then 3)

Q7- child has space maintainer (band and bilateral Band and loop (if the tooth
loop ) in the place of tooth #74, and has number is not present in question ,if
extracted tooth #84, what is the tt ? present it will be lingual arch)

Q8- pt came to the clinic with broken


complete denture, and you want to send it
to the lab to fix it, how should you clean it with sodium hypochloride for 15min
before sending it to the lab?

Q9- a same question but with calculus on Ultrasonic scaler at low speed
the denture ?

Q10- what is the material used for the


major connector in RPD ? -CO-CR

Q11- pt with class2 kennedy classification,


and the last tooth is 2nd premolar which
has a mesial undercut , what should you Round wrought wire clasp with round
use? cross section
Q12- a question about cleft lip and palate ( ican't From 1st branchial arch ( ‫اجاﺑه علي سؤال شﺑهه ولكن لم‬
(remember (‫نتذكر اﻻختيارات‬

Q13- During (RCT or cavity preparation) of #46 , pt


was givin Id nerve block and still feels pain in mesial
aspect ?of the tooth, what to do? The lower molars may receive additional nerve
1-The supply from the nerve to mylohyoid for the pulp
lower anterior teeth cross innervation (Incisive_1 and investing structures
(Nerve crosses the midline from one side to another)
2-The lower premolar may receive additional nerve
supply from the cutaneous coli nerve (branch from
the cervical plexus of nerves) for their labial
mucoperiostieum 3-The lower
molars may receive additional nerve supply from the
nerve to mylohyoid for the pulp and investing
structures

Q14- a
question about the innervations of the tongue ( I
don't remember whether the taste or sensation in Ant 2/3: sensory :lingual branch of V3 from
ant. 2/3 (or post 1/3)? trigeminal nerve . Taste: chorda tympani branch
of facial V11 Post 1/3:
sensory and taste: glossopharyngeal X1

Q15-how many carpules of LA you can five for a child According to weight-----weight *4.4 then divided
(I can't remember his age)? the result on 36 if(2% lidocaine) or 72 if (4%
lidocaine)

Q16- which irrigation solution can kill E-Fesialis? MTAD is more effective than NAOCL in killing
E.faecalis
NAOCL is more effective than MTA in killing E.faecalis
Also, CHX can kill E. faecalis

Q17- first sign of syphilis? Ulcer

Q18- what is the new acceptable name of gingival Gingival overgrowth


enlargement?

Q19- a doctor asked the patient to turn his face to Left sternocleidomastoid muscle
right side and put his hand on the patient's neck to
examine which muscle?

Q20- which type of radiograph can be used to MRI


examine a (sialolith)? (sialography not mentioned in
the choices )
Q21- a question about the histology of salivary gland Serious salivary glands have pyramidal cells with
( one type of cells was described in the question ) rounded nucli

Mucous salivary glands have pyramidal cells


with flatend nucli.

Q22- pt had implant and came after few days with


inflammation and ( I don't remember what was in Perimplantitis with bone loss
radiograph), what is the Diagnosis?

Q23- a picture was given for an edentulous patient Uncomplete question (I haven’t found answer
with two implants in canine areas, and the over about it)
denture became loose , what is the cause?

Q24- a dentist did RCT for one canal in upper 1st 2 canals
premolar, the pt came after few days with pain.
What is the number of canals in the upper 1st
premolar?

Q25- a dentist did RCT for 3 canals in upper 1st


molar, the pt came after few days with pain. What is 4 canals
the number of canals in the upper 1st molar?

Q26- in which root there are two canals in the upper MB root
1st molar?

Q27- what is the prognosis if there is broken


instrument in the middle third and can by bypassed? Good prognosis
Q28- perforation happened during searching for the Put MTA then found the original canal
canal orifice, what should you do?

Q29- a question about pimphigus vulgaris? -------

Q30- pt with history of ischemic heart disease, has ischemic attack RX:
attach during treatment. What should you do? 1-Bronchodialator (Ventolin,Provental)
Then 2- subcutenous epinephrine
Then3- O2 inhalation every 20 min
Q30- pt with history of ischemic heart disease, has
attach during treatment. What should you do? nitroglycerin and oxygen

Q31- asthmatic pt developed attack during tt, what inhalation o2 first ,if not then 100 ml of
slould you do ? hydrocortisone IV .
Q32- you ask the pt to smile and raise the Facial nerve
eye brows , what is the nerve being
examined?
Parathyroid
Q33- what is the gland responsible for
increase ca+ level in blood?

Q35- a swelling on the gingiva interferes incise the lesion and let the tooth erupt
with the eruption of a tooth, what to do?

Q36- a child had trauma on his face. The luxation and laceration
incisor intruded in the bone, and the
superficial layer of mucosa was injured.
What is the Diagnosis?

Q37- pt had trauma and broken incisal


bond the fractured part with composite
angle that doesn’t cause pulp exposure and
brought the broken part with him. What to
do?

Q38- the mechanism of action of LA? chemical

Q39 – a smoker pt with bad oral hygiene generalized chronic periodontitis


and generalized bone resorption, Diagnosis?
Streptococcus-mutans
Q40- which bacteria causes caries ?

Q41- in CEJ ? cementum overlaps enamel


cementum overlaps enamel
cementum just meets enamel
small gap between cementum and enamel

Q42- in pulpitis, vasodilatation of vessels cell rich zone


and -------will happen in which zone of the
pulp?

Q43- (I think 8 yrs old pt ) had trauma and pulpotomy


pin point exposure of the pulp with no pain.
He came to the clinic after 3 days. What
should you do ?
Q46- Pedo P.T with red & inflamed gingiva & excessive salivation?
Ans: Eruption Hematoma

Q47-treatment of head migran?


Ans: Triptans

Q48_treament of status epilepsy?


Ans:

Q49- Treat ment of tonic - colonic epilepsy?


Ans: Valproate
Q49_wt is the meaning of pharmacokinetic of the drug?

Ans: Pharmacodynamics: is the effect that drugs have on the body.

Pharmacokinetics: is the effect the body has on the drugs includes


absorption, distribution, metabolism and excretion of drugs.

Q50_pt with tongue numbness& loss of taste after extraction of


lower 1st molar what is the nerve affected
during local anaesthesia?

Ans: Sensory : Lingual nerve


Taste: Corda tympani

Q51_P.T with renal dialysis have multiple extraction when the


procedure done ?

Ans: One day after dialysis .

Q52-How to anesthetized P.T with truisms?


Ans: Akinosi.

Q53-pt complaining of pain after construction of bridge.


3 unit what is the cause?

Ans: Gap between abutment.

Q54- pregnant women with high risk of caries..asking. can I take


fluoride to prevent ?

Ans : No eve dens about any harm of using fluoride but without
swallowing. #
Q56_dentist finished restoration for rich man he take .
230 SR the restoration usually for normal pnt 85 SR..is it ethical?

Ans: Not ethical

Q57-yrs old bilateral sub mandibular swilling?

Ans: Cherubism .
Note:
Bilateral submandibular swilling :Cherubism
Unilateral submandibular swilling: Fibrous disblagia

Q58-Implant pt have moderate pain post operatively ?

Ans: Ibuprofen 600

Q59-cleft palate due to?

Ans: incomplete union between maxillary process and palatine


arch.

Q60-what is the suitable cement under composite filling?

Ans: CAOH & Zinc phosphate cement


(‫)لو جه خيارات وفي اختيارانك تختار اﻻتنين مع بعض هتختارهم‬

But I f said that The best sutable one it will be CAOH


Q61_how to manage diabetic P.T loss his consciousness in the
clinic?

Ans: Glucagon 1mg

Q62-Usually # no of canals of lower 1st molar?

Ans: 3 Canals

Q63-no.of canals in maxillary 1st premolar ?

Ans: 2 canals

Q64_pt with complete denture complainig from


difficulty in swallowing?

Ans: Posterior over extended

Q65-P.T with truisms the needle used?

Ans: long needle 25 gauge

Q66-fluctuant abscess drain with blade?

Ans: Blade no.11

NoteS: Blade no.11 is using for small incisions, such as


those used for incising abscesses.

Blade no.12 is using for incisions in gingival sulcus


&incisions posterior to the teeth especially in max.tuberosity
area
Blade no.15 is using for flabs and any oral surgaries.

Q67-non maleficence ?

Ans: Don’t harm

Q68-to fill irregular canal l..obdurate wtih


1.vertical condensation?

Ans: Obtura 3

Q69- Def of Dental Caries?

Ans: Dental caries is classified according to Sturdevant,WHO,&


Shafer ,Hine &Levy to:
Q70-during fabrication of crown dentist do buccal grooves?

Ans: to prevent movement during seating.

Q71-picture about attrision


Q72-pic. about Broxism

Q73_43 yrs old complaining from severe mobility of the


..lower incisor on radiograph show high bone loss, what is your
Diagnosis?
Ans: Periodontitis.

Q74_ pic sephalometric analysis pnt with crowded lower


jaw ANB +5, What is your diagnosis?

Ans: class 3 with skeletal class2

Q75_cephlometric case ANB -2, What is your Diagnosis?


Ans: Class3 with skeletal class 3
Q76_ hyertensive pt with 75 mg aspirin want to extact.
lower6?
Ans: Extract if INR below 3.5

Q77-discolerd fractur denture how to clean it?


Ans: Put in NACL for 15 min then clean with soft brush

Q78- radiograph shows sun rays appearance?


Ans: Osteosarcoma.
Q79_ anaesthesia 2% lidocaine HCL with epinephrine
1:50000, what is the colour of carpale?
Ans: Green

Note:
-Lidocaine 2% with Epinephrine 1:100,000 Red.
-Lidocaine 2% with Epinephrine 1:50,000 Green.
-Lidocaine Plain Light Blue.
-Mepivacaine 2% with Levonordefrin 1:20,000 Brown.
-Mepivacaine 3% Plain Tan.
-Prilocaine 4% with Epinephrine 1:200,000 Yellow.
-Prilocaine 4% Plain Black.
-Bupivacaine 0.5% with Epinephrine1:200,000 Blue.
-Articaine 4% with Epinephrine 1:100,00 Gold.
-Articaine 4% with Epinephrine 1:200,000 Silver.

Q80-flouride toxicity?
Ans: 5 mg/kg

Q81-nerve anesthetized lower canine?


Ans: Incisive nerve
Q82- P.T take fluoride from water & food and large dose tablet this
cause?
Ans: systemic fluorosis
Q83-P.T take 15 mg corticosteroid needs extraction?
Ans: 30mg in the day of extraction ‫ﺿعﻒ الكميه‬

Q84-Instrument separation in apical 1/3?


Ans: Retrieval By schartwez protriver B

Q85- Large exposure in 7 years P.T during class 1 cavity


preparation?
Ans: Pulpotomy

Q86- Needle stick injury to dentist not vaccinated the P.T


+VE Hepatitis B, the dentist should to?
Ans: take vaccine and antigen

Q87-3 types of hepatitis transmitted with blood stream?


Ans: Hep-B, Hep-C &Hep-D
Q88-mussle elevate mandible ? Ans: Masseter& temporalis
Q89-X-ray impacted supernumerary teeth?
Ans: Cledocranial dysplasia

Q90-Characteristics of Down’s syndrome?


Ans: People with Down's syndrome often have certain physical
characteristics. Not everyone will have all of them, but they may
include:
 floppiness (hypotonia)
 small nose and flat nasal bridge
 small mouth with a tongue that may stick out
 eyes that slant upwards and outwards
 a flat back of the head
 broad hands with short fingers
 their palm may have only one crease across it
 below-average weight and length at birth
But people with Down's syndrome don't all look the same – they
also look like their parents and family.

Refrance: http://www.nhs.uk/Conditions/Downs-
syndrome/Pages/Symptoms.aspx

Q91-how to anesthetize upper 2nd premolar?


Ans: Middle SAN
Note: Technique of Middle SAN & procedure:
Q92- during preparation for endo pt..lower6 ‫السؤال ناقص‬

Q93- the dentist try to found MB canal.. He makes lateral


Perforation how to manage ?
Ans: MTA then find MB canals

Q94- case about pyogenic granuloma.

Q95-case about facial nerve paralysis?


Q96- where does the cell differentiation into ameloblasts and
odontoblasts occur during tooth formation:
a- at epithelial diaphragm b- at incisal end c- at CEJ
Ans : at CEJ

Q97- patient 5 years with fluoridated water .8 and take fluoride


supplement 0.5
A.Very Mild flourosis
B_Mild fluorosis
C. -Moderate fluorosis
C-.Moderate fluorosis

Ans: Very mild Flourosis


Q98-6 yrs old boy got a small pox to his primary incisors and the
doctor stated that there will be interruption in calcification. at which
stage it will happen
:
a- Histodifferentiation b- morphodifferentiation
Ans : A ( not sure)

Q99- Pt. needs complete denture u take impression with irreversible


hydrocolloid( alginate )& poured it after late more than 15 min. the
cast appears short & chalky the reason is Dehydration of the
impression (not sure)

Q100- patient 5 years with fluoridated water .8 and take fluoride


supplement 0.5
A_.Mild fluorosis
B_.Severe fluorosis
C-.Moderate
D-very mild Flourosis
Ans: Very mild Flourosis
(According to the table in the same question no97)

Q101- Implant patient has moderate pain postoperatively,, you


subscribe Opioid ?
Ans: Acetaminophen 625 mg + 30 mg Codeine 3mg (because opoid
is given in question)
Q102-Component of gatta percha ?
Ans: 20% Gp & 70% ZOE

Q103-The least effective irrigation against E. Facials?


Ans: CHX

Q104- Which of these canal irrigation is UNABLE to


kill E. Facials?
A. NAOH
B.CHX
C.MTAD
Ans: NAOH

Very Important Note:


Sodium hypochlorite= NAOCL not NAOH
MTAD is more effective than NAOCL in killing E.faecalis
NAOCL is more effective than MTA in killing E.faecalis
Also, CHX can kill E. faecalis

Q105- Injury to which nerve for gag reflex?


Ans:
Sensory: Glossopharyngeal nerve
Motor: Vegas nerve
Q106- Treatment of middle root fracture?
Ans: RCT

Q107-- Treatment of static epilepsy ?


Ans:

Q108- psilocin 4% with epinephrine 1:200000?


Ans: Yellow
Q109- Bupivacaine 0,5% with epinephrine 1:200000?
Ans: Blue
Q110- Which brushing technique is recommended for temporary
cleaning following periodontal surgery?
Ans: Charter’s technique

Q111-Which of the following tooth brushing techniques is


advised during post periodontal plastic surgery and in cases of
recession?
a. Bass technique.
b. Fones technique.
c. Leonard technique.
d. Modified Stallman’s technique

Ans: Modified Stallman’s technique

Q112-brushing technique in which parts of the bristles are activated


?
Ans: Bass

Q113- Brushing method in which side parts of bristles are


activated?
Ans: Charter
Q114- /case described and diagnosis is most likely sialadenitis but
patient has allergy to Iodin containing,what kind of diagnostic
image taken:
a- normal Solography
b. CT
c. MRI
d. Occlusal x-ray
Ans: MRI

Q115- /Def of Demonstration?

Q116-Anesthesia for first molar?

Q117-THE Most Important feature of luting cement?


a.tensile steringth
b.comprehensive steringth
c. resistance to solubility
Q118- Child come to clinic with total reverse of upper anterior teeth
sometimes child discease his chin & lateral cephalometric give this
results Sna 80
Snb 82 Anb -2 what is diagnosis?
A-Class I malocclusion with skeletal class III
B-class III malocclusion with skeletal class I
C-class III malocclusion with skeletal class II
D-class III malocclusion with skeletal class III

Ans: Class III malocclusion with skeletal class III

Note: As Sna 80 , Snb 82, so And is -2 , if it is -2 ,it will be


skeletal 3, & asit revers upper anterior , it will be malocclusion
class III

Q119- Pt loss teste in anterior part of tongue due to injury in


a. Facial nerve
b. Mental nerve
Ans: Facial nerve

Q120-Female pt. Comes with endo treated upper central with M &
D caries and have incisal abrasion. Porcelain veneer is planned
with modification to cover incisal edge. Veneer should end:
a. Fourth lingual 0.5 mm before centric occlusal.
B. Fourth 1.5 mm before centric occlusion
c. Fifth 1.5 mm before centric occlusion
Ans: Fourth 1.5 mm before centric occlusion
Q121-the recurrent infection in diabetes is due to the destruction of
which cell:
a. Neutrophils
b. Leukocyte
c. Macrophages
d. Immunoglobulin
Ans: Neutrophils

Q122-Pleomorphic adenoma its size is 1.5 x1.5 cm on . posterior


part of hard palat, what is ttt:
a. Enucleation only
b. Radiotherapy
c. Chemotherapy and enucleation
d. Resection of periosteum
Ans: Resection of periosteum

Q123-Paranasal fluid occurs in all fracture except zygomatic


fracture

Q124-Posterior 1/3 of tongue nerve supply?


Ans: Glossopharyngeal nerve

Q125- After ext lower second molar P.T loss of sensation due to
injury in?
Ans: Ant 2/3 Lingual
Post 1/3 glossopharyngeal nerve
Q126-Cod of anaesthesia? ‫السؤال ناقص‬

Q127-Which Bacterial cause of acute periapical periodontitis?


Ans: Streptococcus Mallari

Q128-Cell in chronic inflammatory?


Ans: Lymphocytes

Q129-Histology of Congental syphilis?


Ans: Histology of congenital syphilis :
- enlarged hyper cellular villi
- Proliferation fatal vascular changes
- Acute or chronic villitis
- plasmocytes.

Q130-Maximum distance for lingual holding arch?

A.2
B.3
C.4.
D.5
Ans: C.4
Q131- patient with eroded palatal surfaces of upper anterior teeth
what's the possibility reason?
a. Peptic ulcer
b. anorexia nervosa
Ans: Peptic ulcer

Q132- patient female 38 y old come to u for check up,with


radiograph u found multiple radiolucency in
?anterior area periapical, no caries no pain wt cause
A. periapical cemental dysplasia
B. fibrous dysplasia
C. apical periodontitis
Ans: A. . periapical cemental dysplasia

Q133-Medication cause gingival enlargement ?


Ans: phenytoin

Q134-two questions with images for kennedy classes ? ‫السؤال ناقص‬

Q135- patient came to you and you give him inf nerve block and he
came to u the day after procedure with trismus from anethesia
which space was affected
a-sub mandibular
b-sub massetric
c. ptergomandibular space
Ans: . ptergomandibular space
Q136-patient has mild stain by tetracycline ,what will you do?
a-home bleach
b-micro abrasion
c-polishing Ans: Home bleach

Q137-patient with badly decayed upper left central with swelling


buccal to the buccal vestibule what is the x-ray , you will do :
a-periapical
b-panorama
Ans: periapical

Q138-epinephrine in endo surgery with anaesthesia


a-prolong time
b-vasodilator to the tissue
Ans: prolong time

Q139- pic with anaesthesia technique of inferior nerve block ?


Q140yrs child with swelling due to badly decayed lower right c with
no crowding
a-extract canine
b-extract the two canines
Ans: extract canine

Q141-What is most favorable place for streptococcus mutants ?


a-pits and fissure
b-proximal
Ans: pits and fissure

Q142-Lefotr 1 injury
a/greater platine artery
b/infra orbital artery
c/maxillary artery
Ans: Greater platine artery

Q143-In order to activation of periodontal instruments the blade


should make angle with facial surface of the tooth:
a- 45:90
b- 90:180
Ans: 45:90

Q144-Most retentive crown is?


a. Full metal
b. Veneer
Ans: Full metal
Q145-yrs child has trauma cause intrusion
a-extraction
b-dont touch it for comfortable
Ans: according to tooth if permanent or diseduous

Q146- dentist at the end of the day want to pour alginate impression
quikly how can he do that
a-increase powder/water ratio
b-hot water
c-slurry water
Ans: Hot water

Q147-chair time relining of denture


a-wax
b-light cure acrylic
c->>>>
Ans: a. wax b. light cure acrylic ‫السؤال ناقص‬

Q148-patient with prothetic valve before surgery give pt


prophylactic AB
Q149-Treatment of kerato cyst ?
Ans:

Q150- 5 years child swallow bottle of mouth wash contain 13mg of


fluoride?
a-refer to emergency immediately
b-make him drink milk or something I don’t remember contain Ca

Ans: make him drink milk or something I don’t remember contain


Ca
Q151- patient can’t localize the pain and you decided to do inter-
ligament injection what is the first place you will inject?
a. Upper Anterior
b. Lower Anterior
c. Upper Posterior
d. Lower Posterior
Ans: Upper Posterior

Q152-X-ray of parotid gland if P.T has iodine sensitivity


A sialogram
B MRI
C CT
Ans: MRI

Q153-long story about blade no.of stap incision for abscess


drainage?
a.11
b.12
c.15
Ans: 11
Q154- Pic. of lower complete over denture asking about. decrease
retention ‫السؤال ناقص‬

Q155_ pic.of diabetic patient with xerostomia


a. caries
b.erosion
c.attrition
Ans: caries

Q156-non carious lesion may related to occlusal defect attrition

Q157-X-ray of 22y old with midline shift in lower jaw with history of
jaw lesion removal ? _
a .dentegrous cyst
b.ameloblastoma
c.odontogenic keratocyst
d. Tumor
Ans: Ameloblastoma

Q158- Management of cardiac P.T in clinic with chest pain?


a. Nitro-glycerine
b. Other medication
Ans: Nitro-glycerine
Q159-long history of patient with notched central at which stage
syphilis affect teeth ?
a. initiation
b .proliferation
c. histodifferentiation
d. morphodifferentiation

Ans: morphodiffierentiation

Q160-Long story asking about flap retractor?


Ans: Minnesota

Q161- something about dead space


a.150
b.250
c.350
d.450‫?????? السؤال ناقص‬
Ans: 150

Q162- Anaesthesia for P.T with amide allergy?


_lidocaine
_ prilocaine
_ mepevicaine
Ans:----(All are from amid family)
Q163-pt of mandible ask about suitable NO.# of implant regarding
cost ?
a. 2
b. 3
c. 5
Ans: 2

Q164-Muscle of mouth opening?


A. masseter
b. digastric
c. L.pteryogoid

Ans: Digastric

Q165- down on upper permanent lateral and tooth broken close


pulp without exposer. What we will do to....him
Ans: Indirect Pulp capping
Q166- Another patient fell down on 22 but the pulp hasn't exposed
and tooth has changed its colour so what's the procedure
Ans: RCT

Q167- Patient came with swelling in one side of his face with pain
increase during eating
Ans: salivary gland stone (sialolythiasis)

Q168-A patient with chronic renal disease when we could treat him
in the dental clinic?
A.One day.
b.Two days
c. a month
d. A week
Ans: one day

Q170-Patient came with hepatitis and we have done a procedure to


him and another patient came after him , does regular disinfectant
enough for the clinic?

Q171-What's the most difficult tooth to extract


a.mandibular third molar
b.mand first premolar
c.max first premolar
d.max canine
Ans: mand third molar
Q172- Best media for avulsed tooth?
a. low fat milk
b.saliva blood
c.Normal salin
d.blood

Ans: Low fat milk


Q172- More than something that affect Thumb sucking ?
a.duration
b.age
c.force

Ans: duration

Q173- THE Most Important feature of luting cement ?


a. Tensile steringth
b. Comprehensive steringth
c. Resistance to solubility
Ans: Resistance to solubility

Q174- Gow -Gates anaesthesia


Q175-Def of Cementoblastoma ? Def I:

Or Def II:
Q176-intra ligamentary injection:
A- incease the pulpal circulation for 30 min
B-decrease pulpal circulation
C-increase ‫السؤال ناقص‬

Q 177-lateral multilocular cyst: Botryoid odontogenic cyst ‫السؤال‬


‫ناقص‬

Q178-What is the manifestation taken by a noxious stimulus?


A. Pain
B. Pain threshold
C. Pain manifestation
D. Pain perception
Ans: Pain threshold

Q179- which one is used as vitality test


A. Doppler fluorescence
B. heat
C. don’t remember
Ans: Heat
Q180- picture of patient with Complete denture , the patients isn't
satisfied, knowing that he was satisfied with the colour of the teeth,
what is the most probable mistake done by the doctor?
A-incorrect mesio-distal angles
B. shifted midline
C. incorrect canine line
D. Don’t remember
Ans: shifted midline

Q181-50 years old female have paroxysmal unilateral short pain


which aggravates on touching atypical odontology :
A. Atypical neuralgia
b. Trigeminal neuralgia
Ans: Trigeminal neuralgia

Q182- Which fluoride can be applied professionally by the dentist?


1. Act
2. varnish
3. Gel n
4. Prevident
Ans: Varnish

Note :But if duraphat is present choose it because it is more


popular...
Q183- what is used with PPR
A. GIC
B. flowable composite
C. Don’t remember
Ans: flowable composite
Q184-patient have to do complete denture, but have few remaining
teeth with bad perio health that have to be extracted in the future,
but not now, which denture has to be done:
A.interim denture
B-transfert denture
C-dont remember ‫السؤال ناقص‬

Q185_ At the beginning of the operation day in the clinic you


should use water air spray for 3 min. for which Microorganism
aeruginosa
Ans: Streptoccous salivaries

Q186-diagnodent can detect


A.deep caries
B.surface caries
C.2other options
Ans: surface caries

Q187-8years old child came to the clinic with badly decade lower
temporary mandibular canine, pain and swelling, what to do:
a. Extract the canine and space maintainer
b. Extract both deciduous canine

Ans: Extract both deciduous canine


Q188- 35 years old patient came to the clinic, molar with severe
pain, fluctuent swelling. First thing to do as emergency treatment:
A. incision and drainage
B. incision and antibiotics
Ans: incision and drainage

Note: Incision and drainage because the AB doesn't take effect


while abscess is closed must open it first to relief pain then write the
AB

Q189-RCT patient has severe pain on 25 with cold that stays few
minutes, and spontaneous pain ,positive on probing , but no pain on
percussion
A.reversible pulpitis
B.irreversible pulpitis
C.dont know until further informations

Ans: irreversible pulpitis

Q190_ patient had composite filling done 2weeks ago, came with
pain on biting he says that the filling was high since the beginning,
vitality test positive, pain on percussion, what is the diagnosis: acute
apical periodontitis
Q191_ Child came to the clinic for a follow up, has done
2bitewing 6 months ago with no caries, which X-ray
survey has to be done for him
A.panoramic and bitewings
B.2bitewings
C.no need for xray
Ans: no need for xray

Q192_ pink discoloration on buccal incisive,x ray shows


radiolucency in the cervical part and the middle part
Ans: internal resorption

Q193_ Child patient take oral sedation before appointment and


present with physical volt what should dentist do:
1. Conscious sedation.
2. Redo sedation.
3. Tie with bamboos board.
4.Tie in unite with bandage
Ans: Conscious sedation

Q194-composite resin with large size filler what is most badly


affected:
a-resin bond
b- hardness
c-polish ability
Ans: polish ability

GOOD LUCK

You might also like